Download as pdf or txt
Download as pdf or txt
You are on page 1of 543

Preparation for ASPRS

Certification: General
Knowledge
Robert Burtch, CP (Ret), PS
Professor Emeritus
Ferris State University
Introduction
Preparation for ASPRS Certification: General Knowledge Workshop
WORKSHOP OUTLINE
• Purpose of the exam and certification
• Geodesy/Surveying principles
• Lidar
• Photogrammetry
• Remote sensing
• Geographic Information Systems
• UAS
• Other issues pertaining to the exam process
Purpose of Workshop What Will Not be Covered

• Explain topical areas one • Theoretical background to teach


can expect to appear on the participants on topics that
exam will be found on the exam
• Exam questions from the bank of
• Provide participant with questions maintained by ASPRS
understanding what they or other exam-writing body
need to study in preparing • Need to preserve integrity of
for the exam the exam process
• Help participants
understand how to take the
exam
PURPOSE OF CERTIFICATION/LICENSURE
• Protect the health and safety of public
• Provides measure of minimum competence or qualifications of a
professional for a potential client
• Helps to codify a professional code of ethics that the
licensee/certified professional subscribes to
• Practitioner recognition
• Knowledge advancement
• Etc.
Certification Versus Licensure
• ASPRS Certification Program
• Official recognition by one’s colleagues/peers
• Demonstrate professional integrity & competence
• Considered “specialty certification”
• Not substitute for licensure
• Licensure
• Legal act – police power
• Protect public health, safety, welfare
Certification Program Objectives
• To identify and recognize those persons who, after careful, just
appraisal by their peers, and after passing a written examination are
considered to have met the requirements established by the Society
for certification.
• To provide a basis for weighing the validity or allegations and
complaints that involve practicing photogrammetrists and mapping
scientists, and for taking appropriate action in connection therewith.
• To encourage persons as yet not fully qualified to work towards
certification as a goal of professional achievement.
• To encourage certified persons, through the recertification process, to
continue their professional achievements as the rapid change in
technology occurs.
ASPRS Code of Ethics
• Honesty, justice, and courtesy form a moral philosophy which, associated with mutual
interest among people, should be the principles on which ethics are founded.
• Each person who is engaged in the use, development, and improvement of the mapping
sciences (Photogrammetry, Remote Sensing, Geographic Information Systems, and
related disciplines) should accept those principles as a set of dynamic guides for conduct
and a way of life rather than merely for passive observance. It is an inherent obligation to
apply oneself to one’s profession with all diligence and in so doing to be guided by this

Each person in the mapping sciences profession


• Achieve excellence in practice
• Maintain high standards and ethics
• Conduct themselves professionally to an employer, all clients, colleagues and associates,
and society at large.

https://www.asprs.org/ASPRS-Organization/code-of-ethics-of-the-
american-society-for-photogrammetry-and-remote-sensing
1. Rules that govern professional conduct are referred to as
_______________.
1. Rules that govern professional conduct are referred to as
____ethics_ _____.

From Ed Bristow (Side Shots,


www.flatironsinc.com/pdfs/EthicsArticle.pdf)
Ethics are “those conscious guidelines that allow human
https://www.standardsuniversity.o

beings to live in harmony as a society, resulting in


rg/wp-content/uploads/ethics-

prevention of personal gain at the expense of and/or


intentional harm to individuals”
technology.jpg
2. A violation of a professional code of ethics can
lead to
a) arrest by the prosecuting attorney.
b) expulsion from the professional society
c) the inability to practice in the profession.
d) All of the above.
• Code of Ethics is not a legal document
• Generally will not mean one cannot continue to practice in that profession
• Code of Ethics does not create new moral or ethical principles – these principles
rooted in centuries of societal & human interactions

2. A violation of a professional code of ethics can


lead to
a) arrest by the prosecuting attorney.
b) expulsion from the professional society
c) the inability to practice in the profession.
d) All of the above.
3. A GIS professional was charged with preparing a map showing
abandoned utilities in an area to be cleared for a new development.
In order to save money for the client, utility companies in the area
were not contacted. Instead the professional relied on existing data
that was recently compiled in an adjacent property. During
construction a gas line was ruptured and service was disrupted in the
community. The GIS professional was subsequently accused of
_________________.
a) defeasance
b) malfeasance
c) misfeasance
d) nonfeasance
3. A GIS professional was charged with preparing a map showing
abandoned utilities in an area to be cleared for a new development.
In order to save money for the client, utility companies in the area
were not contacted. Instead the professional relied on existing data
that was recently compiled in an adjacent property. During
construction a gas line was ruptured and service was disrupted in the
community. The GIS professional was subsequently accused of
_________________.
a) defeasance
b) malfeasance
c) misfeasance
d) nonfeasance
From: https://legal-dictionary.thefreedictionary.com

Ethics
• Defeasance – antiquated word for a document which terminated the
effect of an existing writing such as a deed, bond, or contract if some
event occurs.
• Malfeasance – intentionally doing something either legally or morally
wrong which one had no right to do.
• Misfeasance – management of a business, public office or other
responsibility in which there are errors and an unfortunate result
through mistake or carelessness, but without evil intent and/or
violation of law.
• Nonfeasance – failure of an agent (employee) to perform a task
he/she has agreed to do for his/her principal (employer)
4. Western Resources is an OEM who provide geospatial equipment to
Environmental Resources Inc. (ERI). Western provided ERI with a high-end
piece of data collection equipment which ERI returned because it did not
meet ERI’s specification. In a meeting with Bill Estes, Western’s
representative, Al Osgood, ERI’s representative, and Gene Smyth,
Western’s engineer who has intimidate knowledge of how the equipment
works. Bill Estes claims that the equipment is correct but during the
meeting Gene Smythe feels that the equipment was not properly tested
and failed due to internal problems. Gene Smyth should
a) affirm Bill Estes’ assertion that the equipment is correct.
b) keep quiet since Bill Estes is Western’s representative.
c) state his concerns to all during the meeting.
d) state his concerns to Bill Estes after the meeting was over.
Adapted from http://engineering.Missouri.edu/Mac/files/engineering_ethics1.pdf

4. Western Resources is an OEM who provide geospatial equipment to


Environmental Resources Inc. (ERI). Western provided ERI with a high-end
piece of data collection equipment which ERI returned because it did not
meet ERI’s specification. In a meeting with Bill Estes, Western’s
representative, Al Osgood, ERI’s representative, and Gene Smyth,
Western’s engineer who has intimidate knowledge of how the equipment
works. Bill Estes claims that the equipment is correct but during the
meeting Gene Smythe feels that the equipment was not properly tested
and failed due to internal problems. Gene Smyth should
a) affirm Bill Estes’ assertion that the equipment is correct.
b) keep quiet since Bill Estes is Western’s representative.
c) state his concerns to all during the meeting.
d) state his concerns to Bill Estes after the meeting was over.
Ethics
• Gene’s silence at the meeting is probably appropriate
• He is technical support not public relations
• His suspicions were not yet confirmed
• A preliminary contradiction of Bill Estes’ statement was unwarranted
• Raising his objection to Bill after the meeting is appropriate.
Question
• Provide examples of the following:
• Conflict of interest,
• Professional misconduct
• Professional misrepresentation
• Professional malfeasance
5. Ken Roberts, Director of Mapping for Central Geomatics, is offer a
new position with Ajax Aerial Survey as a Vice-President, a position
he was passed over at his current company. Mr. Roberts has
excellent relationships with Central Geomatics’ clients, always
meeting their expectations. He should tell his client contacts
a)he can still meet their future mapping needs at Ajax Aerial
Survey.
b)he is now working at Ajax Aerial Survey and would appreciate
their work.
c)he was passed over for promotion at Central Geomatics and is
leaving because of that.
d)nothing.
5. Ken Roberts, Director of Mapping for Central Geomatics, is offer a
new position with Ajax Aerial Survey as a Vice-President, a position
he was passed over at his current company. Mr. Roberts has
excellent relationships with Central Geomatics’ clients, always
meeting their expectations. He should tell his client contacts
a)he can still meet their future mapping needs at Ajax Aerial
Survey.
b)he is now working at Ajax Aerial Survey and would appreciate
their work.
c)he was passed over for promotion at Central Geomatics and is
leaving because of that.
d)nothing.
History
• 1975 – Certified Photogrammetrist
• 1992 – Certified Mapping Scientist in Remote Sensing and GIS/LIS
• Certification achieved by peer review
• 1998 – Certification required peer review and successful passing exam
• 2003 – Technician Certification Program in photogrammetry, remote
sensing, GIS/LIS
• 2006 – Provisional Certification Program
• 2013 –Received accreditation from the Council of Engineering & Scientific
Specialty Boards
• 2013 – Provisional Certification Program renamed Geospatial Intern
Certification Program
• 2015 – Professional and Technician Certification in lidar
• 2017 – Professional and Technician Certification in UAS
PROFESSIONAL CERTIFICATION
REQUIREMENTS
• Six years of professional experience as approved by peer group
• Degree in engineering, or in natural or physical sciences may be partially
applied to experience – ½ year for BS, for MS, for PhD
• See program for specific requirements for each certification
• References (4)
• Declaration of compliance with ASPRS Code of Ethics
• Successful completion of written exam
• Active certification for 5 years
TECHNICIAN CERTIFICATION REQUIREMENTS
• 3 years of experience – 2 in the specialty category
• Associate or higher degree – ½ year
• Application with 4 references
• Declaration of compliance with ASPRS Code of Ethics
• Peer review
• Successful completion of written exam
• Active certification for 3 years
Geospatial Intern Certification
• Formerly called “Provisional Certification”
• Photogrammetry, Remote Sensing, GIS, UAS and Lidar
• Professional and technologist levels
• Graduating students in spatial sciences meeting educational/program
requirements
• Take certification exam as they complete studies
• Pass peer review
• Become fully certified upon completion of required job experience
ASPRS CERTIFICATION APPLICATION FEES
TYPE OF APPLICATION ASPRS Non-
Members Members
Initial Certification $300 $450
Recertification $200 $350
Geospatial Intern Certification
1. Initial Application $200 $250
2. Final Application $200 $350
Initial Certification – Technologist $200 $350
Recertification - Technologist $125 $275

Fees are not refundable in event of non-certification


CERTIFICATION EXAM
• Multidisciplinary
• Reflects nature of professional practice
• Understanding ancillary tasks gives professional the ability to evaluate the project needs and
technological capabilities
• 100 questions (50 in Technologist exam)
• Multiple Choice, True/False, Fill-In the Blank, Problems
• 4 hours (2 hours in Technologist exam)
• Allowed one BOUND reference
• Weighted (33%) towards specialty
• Remaining questions test for basic knowledge in spatial sciences
• Exams in allied areas reflect basic conceptual knowledge, not more detailed questions
• Current pass rate 90% (2017)
• Cut score is 75% (2017)
CERTIFICATION PROCESS
• Contacts:
• Jesse Winch - ASPRS headquarters
• jwinch@asprs.org or certification@asprs.org
• ASPRS Web Site:
• https://www.asprs.org/Certification
• Applicant fills out the application form
• Sent to ASPRS
• 4 letters of recommendation also sent to ASPRS by those doing the recommendation (for
Geospatial Intern, need letter from advisor or other faculty member – 3 additional letters for full
certification)
• For Geospatial Intern – copy of transcripts showing at least 2 geospatial courses
• Completed application sent to the appropriate evaluation committee
• Determine whether minimum criteria met
• If approved, applicant is notified, and test proctor identified
CERTIFICATION PROCESS
• Proctor arranges test location and time with applicant
• Proctor sends completed exam to chair of Certification committee for
grading
• Applicant notified after grading
• If applicant fails the exam, they can retake the exam within 6 months

• If certification lapses by more than 1 year – required to take the exam


again.
Exam Matrix By Topic (Professional):
Certified Certified Certified Certified
Certified
Mapping Mapping Mapping Mapping
Focus Area Photogram-
Scientist Scientist Scientist Scientist
metrist

Andelin, https://www.asprs.org/featured/uas-certified-mapping-webinar.html
GIS/LIS Remote Sensing Lidar UAS

UAS information from: ASPRS Certification Webinar by K. Schuckman and E.


Math/Science 13-15% 14-16% 14-16% 10-16% 10-16%
Engineering/
6-7% 5-6% 5-6% 5-6% 5-6%
Surveying
Physics 9-11% 7-9% 9-10% 9-10% 9-10%
Imaging 12-15% 18-20% 25-28% 12-15% 12-15%
Photogrammetr
25-27% 10-13% 10-14% 10-15% 10-15%
y
GIS 12-15% 21-23% 12-15% 10-15% 10-15%
Lidar 4-6% 4-6% 4-6% 25-30% 4-6%
Ethics/General 15% 15% 15% 15% 15%
UAS 25-30%
Exam Matrix By Topic (Technologist):
Certified
Certified Certified Certified
Photogram- Certified GIS/LIS
Focus Area Remote Sensing Lidar UAS
metric Technologist
Technologist Technologist Technologist
Technologist

Andelin, https://www.asprs.org/featured/uas-certified-mapping-webinar.html
Mathematics 10% 10% 10% 10% 10%

UAS information from: ASPRS Certification Webinar by K. Schuckman and E.


Earth Science 4% 4% 4% 4% 4%
Basic Surveying 10% 10% 10% 6% 6%
Physics 4% 4% 4% 4% 4%
Standards 8% 8% 8% 8% 8%
Aerial
12% 12% 12% 6% 6%
Photography
Practice Issues 6% 6% 6% 6% 6%
Photogrammetry 34% 6% 12% 6% 6%
Image
Processing/ 8% 10% 30% 10% 10%
Remote Sensing
GIS/LIS 4% 30% 4% 4% 4%
Lidar 4% 6% 6% 34% 4%
UAS 32%
Recertification
• Certified for life
• Active certification requires recertification
• 5 years Professional; 3 years Technician
• If not recertified – become inactive or retired
• No recertification for Geospatial Intern Certification
• Need 100 hours (PDH) for professional and 60 hours for technician
certification

• If certification lapses by more than 1 year – required to take the exam


again.
Recertification
Evaluation Activity for Recertification Criteria Maximum Number of PDH
Allowed Documentation Requirement
Within Last 5 Years
Minimum Continuing Experience: Applicant has been 3 years of the past 5 years actively 25 PDH Credits Document Position(s)
active in providing services within their certified providing services Description(s) and dates
discipline, or has been in the academic arena involved
directly with those subjects.

Applicant has contributed and participated in activities of See items 1 – 10 listed below for PDH 20 PDH Credits Document Titles with
service to the profession and community (see list below) Credits Referencing

Published Papers and/or professional manuals See items 11-12 listed below for PDH 40 PDH Credits Document time spent, titles
(see list below) credits with full referencing

Applicant has attended workshops/classes or instructed 1 PDH per hour attended (0.1 CEU = 1 50 PDH (5 CEU) Document Course, Titles, and
in directly related subjects PDH) Date

Applicant has attended technical conferences and other 1 PDH per hour of technical session 40 PDH Document Meeting Titles and
professional meetings sponsored by ASPRS, ISPRS, attendance Dates
SPIE, and other appropriate professional organizations.

Applicant has actively served on ASPRS Committees. 5 PDH Credits 25 PDH Credits Document Activity and Dates
Officer, Division Chair, Region Officer, Chapter Officer, per Year
or related support. See list below for PDH Credits
Recertification
Professional Activities PDH Credits
(1) Holding office [e.g., committee chair, vice chair, secretary, board trustee] (1 point per year) 5

(2) Committee Member 3

(3) Meeting/Conference Chair or Vice Chair 10

(4) Session Moderator at Technical Conference 5

(5) Author or co-author of a conference technical paper, poster session paper or panel session 3

(6) Adjunct teaching a formal class in an academic setting [when not applicant’s primary occupation](maximum of 5 10
PDH credits per year)
(7) Workshop Instructor (2 PDH credits per course for a maximum of 4 credits per year) 12

(8) Examination Committee Contributor (2 PDH credits per instance for a maximum of 5 per year) 10

(9) ASPRS Certification exam proctor/panel (2 PDH credits per instance for a maximum of 4 per year 8

(10) Community/Career extra curricular, i.e., Pro Bono involvement (geomatics-related), e.g., Public Agency (geospatial) 10
Advisory Board, Citizens Advisory Group, (5 PDH credits per activity)
(11) Author/editor of published geospatial-related book 15

(12) Author (not prime) of one or more chapters of a published geospatial-related book 5
Recertification CEU Tracking Spreadsheet
OTHER LICENSURE/CERTIFICATION ACTIVITIES
• Surveyors and Engineers licensed by state
• Photogrammetry licensure as surveyors
• Florida, North and South Carolina, Oregon
• Virginia, West Virginia
• GIS certification
• GIS Certification Institute (GISCI) administers certification program
• ACSM Hydrographer Certification
• 5 years experience (2 years in responsible charge, 2 years in field
• Examination – 160 multiple choice questions
• Canadian Institute of Geomatics Certification for Geomatics Specialists
• Specialization: Geomatics Specialist (photogrammetry, remote sensing,
GIS/LIS, geodesy, cartography), Geomatics Manager
GISCI – GIS Certification Institute
• Outgrowth from URISA’s certification initiative
• Only programs offered are certification
• Point-based in 3 categories: education, experience, contributions
• Minimum number in each category (30 education, 60 experience, 8
contribution)
• 5 year certification
National Council of Examiners for Engineering
and Surveying (NCEES)
• Comprise engineering and surveying licensing boards
• Responsible for testing of engineers and surveyors
• Establishes Model Law – model for state legislation for licensure of
engineers and surveyors
• Will not create discipline-specific exam unless more than 10 states
require licensure of discipline
• ASPRS working with Colonial States Boards on developing bank of test
questions
“Delegates Debate the Issues at NCEES Annual Meeting”, Licensure Exchange, 20(5), October 2016

NCEES
• New Position Statement
• Remote sensing technologies
• Lidar, photogrammetry, unmanned aircraft systems
• Other emerging technologies
• Should be under responsible charge of licensed professional to safeguard the
public
For Further Information Contact
Robert Burtch PS, CP (Ret.)
robert.burtch66@gmail.com
rburtch@charter.net
Geodesy/Surveying
Principles
Preparation for ASPRS Certification: General Knowledge
1. The reference surface upon which elevations are
measured in the U.S. is officially called the
______________________________________.
1. The reference surface upon which elevations are
measured in the U.S. is officially called the North
American Vertical Datum of 1988.

http://earth-info.nga.mil/GandG/images/whichref.gif
2. A network of sites managed by the National
Geodetic Survey that provides Global Navigation
Satellite System data to users is referred to as
__________________________.
2. A network of sites managed by the National
Geodetic Survey that provides Global Navigation
Satellite System data to users is referred to as
Continuously Operating Reference Station
(CORS)

https://www.e-
education.psu.edu/geog862/node/1830
3. When a GPS signal is captured at the receiver as a
result of two or more different paths, this is
called ________.
3. When a GPS signal is captured at the receiver as a
result of two or more different paths, this is
called multipath.

• solid red lines are the emitted laser pulse and


return - generate a point at the green oval on
the object.
• portion of the signal reflected from the object
hits the ground, and then returns to the
sensor
• incorrect point will be produced at the
orange oval indicated, parallel to the original
pulse emitted.
• algorithms and statistical methods can be
applied to reduce the effects of this
phenomenon.
https://www.hpc.msstate.edu/publications/docs/2016/09/15022LIDAR_V5.pdf
4. A continuous surface that is at all points
perpendicular to the direction of gravity is called
a(n)
a) ellipsoid.
b) geoid.
c) oblate spheroid.
d) terrain surface.
4. A continuous surface that is at all points
perpendicular to the direction of gravity is called
a(n)
http://www.seos-

a) ellipsoid.
project.eu/modules/oceanc
urrents/oceancurrents-c06-
s03-p01.html

b) geoid.
c) oblate spheroid.
d) terrain surface.
5. The line formed by the intersection of the earth’s
surface with a plane that includes the earth’s axis
of rotation is called the
a) geographic meridian.
b) grid meridian.
c) magnetic meridian.
d) rotated meridian.
5. The line formed by the intersection of the earth’s
surface with a plane that includes the earth’s axis
of rotation is called the
a) geographic meridian.
b) grid meridian.
c) magnetic meridian.
d) rotated meridian.
Meridians
https://www.britannica.com/science/latitude

http://www.physica
lgeography.net/fun
damentals/2b.html

http://www.georeference.org/doc/view_grid.htm

Graticules are always expressed in geographic coordinates (latitude and longitude) while grids are
expressed in the native X and Y coordinates of the coordinate system of the component. For
components using the Latitude / Longitude "non-projection", both graticules and grids will appear as a
grid of horizontal and vertical straight lines.

In projected coordinate systems, graticules will be created as curved lines (if necessary) to parallel the
curved form of meridians of longitude or parallels of latitude in the projection. Grids, however, will
always appear as a grid of horizontal and vertical straight lines.
6. The global positioning system measures in a
a) geocentric coordinate system.
b) geodetic coordinate system.
c) local vertical coordinate system.
d) state plane coordinates.
6. The global positioning system measures in a
a) geocentric coordinate system.
b) geodetic coordinate system.
c) local vertical coordinate system.
https://kartoweb.itc.nl/geometrics/Coordinate

d) state plane coordinates. %20systems/coordsys.html


GEODESY PRINCIPLES
• GPS is an earth- • Geodetic coordinates
centered, earth-fixed
system
• While State Plane
Coordinates can be
output from a GPS
receiver, processing has
to be performed to
transform the
measured coordinates
to the SPC system
http://geoengine.nga.mil/geospatial/SW_TOOLS/NIMAMUSE/webinter/geotrans2/h
elp/coordDescriptions.htm
7. A projection onto a tangent plane, where the projection
lines are parallel to each other and perpendicular to the
tangent plane is a(n)
a) conical projection.
b) cylindrical projection.
c) gnomonic projection.
d) orthographic projection.
e) stereographic projection.
7. A projection onto a tangent plane, where the projection
lines are parallel to each other and perpendicular to the
tangent plane is a(n)
a) conical projection.
b) cylindrical projection.
c) gnomonic projection.
d) orthographic projection.
e) stereographic projection.
GEODESY PRINCIPLES
• Orthographic projection
• If tangent point is located at one of poles then each parallel
shown correctly to scale at a circle
• Distanced between circles gets smaller from center of map
• Azimuthal projection – azimuth from tangent point correct
GEODESY PRINCIPLES
• Gnomonic projection
• Plane tangent to sphere with projection from center of sphere
• Great circles shown as straight lines
• Intersection of a plane with the sphere containing the center of the sphere
• Shapes and size have extreme distortion
• Azimuthal projection
• Not possible to map entire hemisphere
GEODESY PRINCIPLES
• Stereographic projection
• Similar to gnomonic except projection done at opposite
point from tangent on sphere
• Azimuthal and conformal
• Scale increases as we move away from tangent point
GEODESY PRINCIPLES
• Conical projection • Cylindrical projection
• Use cone as projection • Use cylinder as projection
surface – developable surface – developable
into plane into plane
State Plane Coordinates
GEODESY PRINCIPLES
• GPS is an earth- • Geodetic coordinates
centered, earth-fixed
system
• While State Plane
Coordinates can be
output from a GPS
receiver, processing has
to be performed to
transform the
measured coordinates
to the SPC system
http://geoengine.nga.mil/geospatial/SW_TOOLS/NIMAMUSE/webinter/geotrans2/h
elp/coordDescriptions.htm
8. The most accurate method of transforming NAD
27 coordinates to NAD 83 is by
a) adding the published datum shifts to the
NAD 27 coordinates.
b) adjusting the original measurements to
the NAD 83 datum.
c) developing a local affine transformation.
d) using NGS’ NADCON or USACE’s
CORPSCON software.
e) using the Molendesky transformation.
8. The most accurate method of transforming NAD
27 coordinates to NAD 83 is by
a) adding the published datum shifts to the
NAD 27 coordinates.
b) adjusting the original measurements to
the NAD 83 datum.
c) developing a local affine transformation.
d) using NGS’ NADCON or USACE’s
CORPSCON software.
e) using the Molendesky transformation.
GEODESY PRINCIPLES
• The correct answer is b) Adjusting the original observations
• All other methods approximations
• Example: Station Spicer, Michigan South Zone
• NAD 27: λ =85o36’06.8929” φ= 43o40’38.53246”
• CORPSCON NAD 83: N=242,600.795m E=3,900,389.604m
• NGS published NAD 83: N=242,601.045m E=3,900,390.068
• Difference: N=0.250m E=0.464m
9. The process of determining differences in
elevation between points at some distance from
each other by use of a surveyors’ level together
with a graduated measuring rod is referred to as
a) barometric leveling.
b) differential leveling.
c) reciprocal leveling.
d) trigonometric leveling.
9. The process of determining differences in
elevation between points at some distance from
each other by use of a surveyors’ level together
with a graduated measuring rod is referred to as
a) barometric leveling. http://www.colorado.edu/geography/courses/geog_2043_f01/lab01_4
.html

b) differential leveling.
c) reciprocal leveling.
d) trigonometric leveling.
Leveling

http://www.slideshare.net/pr
ashantsingham/levelling-
survey

https://engineering.purdue.edu/~asm215/topi
cs/difflevl.html

http://nationalmap.gov/ustopo/
http://encyclopedia2.thefreedictionary.c photos/j3-barometer.jpg
om/leveling
10. A distance is measured as 146,251.66 meters.
Using the U.S. Survey Foot conversion, what is the
distance in feet?
a) 44,577.51’
b) 44,577.60’
c) 479,827.32’
d) 479,828.28’
10. A distance is measured as 146,251.66 meters.
Using the U.S. Survey Foot conversion, what is the
distance in feet?
a) 44,577.51’
b) 44,577.60’
3937 𝑓𝑓𝑓𝑓
c) 479,827.32’ 146,251.66 𝑚𝑚
1200 𝑚𝑚
= 479,827.32′

d) 479,828.28’ 1" 1′ 100 𝑐𝑐𝑐𝑐


146,251.66 = 479,828.28′
2.54 𝑐𝑐𝑐𝑐 12" 1 𝑚𝑚
Foot-Meter Conversion
• U.S. Metric Law of 1866
• Allowed for metric measures to be used in U.S.
• Set standard of 1m = 39.37” (exact)
• Signed by President Andrew Jackson on July 28, 1866
• Mendenhall Order
• Imperial standard yard used for calibration becoming unstable and shortening
• Office of Weights and Measures using the metric standard for comparisons
• Order stated that Metric Law of 1866 was satisfactory ( 1yd = 3600/3937m - Exact)
• International yard conversion
• Adopted in U.S. on July 1, 1959
• 1’ = 25.4mm (exact)
11. The height of a point on the terrain above the
geoid is called the
a) ellipsoid height.
b) geocentric height.
c) geoidal height.
d) orthometric height.
11. The height of a point on the terrain above the
geoid is called the
a) ellipsoid height.
b) geocentric height.
c) geoidal height.
d) orthometric height.

http://mycoordinates.org/the-missing-observation-orthometric-
height /
GEODESY PRINCIPLES
• Orthometric height (H) -- this is the elevation commonly
used in surveying and mapping
• Ellipsoid height (h) is height above the ellipsoid
• Geoidal height (N) – separation between ellipsoid and
geoid

http://onlinemanuals.txdot.gov/txdotmanuals/ess/surveying_vertical_networks__with__gps.htm
GEODESY PRINCIPLES
• Geoidal height is distance between the geoid and ellipsoid
• Also called the geoidal undulation or geoidal separation

• Note: some authors call the geoid height the elevation of a terrain
point above the geoid (orthometric height)
12. An error that follows a definite pattern and
whose magnitude and size can be determined is
referred to as a(n)
a) accidental error.
b) blunder.
c) random error.
d) systematic error.
12. An error that follows a definite pattern and
whose magnitude and size can be determined is
referred to as a(n)
a) accidental error.
b) blunder. https://www.e-education.psu.edu/natureofgeoinfo/c5_p5.html

c) random error.
d) systematic error.
13. A least squares adjustment
a) provides unbiased estimates of
unknown parameters.
b) results in the correct or true value of
the unknown parameters.
c) solves for the unknown parameters in a
unique manner.
d) takes observed quantities and adjusts
them to fit a normal distribution.
13. A least squares adjustment
a) provides unbiased estimates of
unknown parameters.
b) results in the correct or true value of
the unknown parameters.
c) solves for the unknown parameters in a
unique manner.
d) takes observed quantities and adjusts
them to fit a normal distribution.
LEAST SQUARES
• Yields most probable value
when the observations are
equally weighted
• Most probable value of a
quantity is the specific value
for which the sum of the
squares of the errors is a
minimum
• Best statistical estimates of
the parameters
14. Differential GPS eliminates
a) atmospheric errors.
b) multipath errors.
c) orbital errors.
d) satellite clock errors.
14. Differential GPS eliminates
a) atmospheric errors.
b) multipath errors.
c) orbital errors.
d) satellite clock errors.

http://celebrating200years.noaa.gov/transformations/mappi
ng/image4.html

DGPS can reduce the effects of atmospheric errors and


orbital errors, but does not eliminate them.

DGPS does not affect multipath errors.


GPS Errors
• Satellite errors http://www.blackboxcamera.com/pic-osd/gps_limits.htm

• Orbital errors
• Satellite clock errors
• Propagation errors
• Ionospheric errors
• Tropospheric errors
• Receiver errors
• Receive clock errors
• Multipath
• Receiver noise
15. A __________ of a route location is the same as if
we were in an aircraft looking straight down.
a) cross section
b) plan view
c) profile
d) reciprocal level run
15. A __________ of a route location is the same as if
we were in an aircraft looking straight down.
a) cross section
b) plan view
c) profile http://engineeringtraining.tpub.com
/14071/css/14071_207.htm

d) reciprocal level run


16. A level surface is a ____________.
a) curved surface where every point is
parallel to the plumb line
b) curved surface where every point is
perpendicular to the plumb line
c) straight surface where every point is
normal to the horizon
d) straight surface where every point is
parallel to the horizon
https://gradeup.co/levelling-i-2f5ad940-b7b7-
11e6-a89b-4293cf7f136d
16. A level surface is a ____________.
a) curved surface where every point is
parallel to the plumb line
b) curved surface where every point is
perpendicular to the plumb line
c) straight surface where every point is
normal to the horizon
d) straight surface where every point is
parallel to the horizon
17. The zenith is the point directly
a) above the North Pole.
b) above the South Pole.
c) above the Equator.
d) above the observer.
17. The zenith is the point directly
a) above the North Pole. http://docs.autodesk.com/CIVIL/2010/ENU/AutoCAD%

b) above the South Pole.


20Civil%202010%20User%20Documentation/index.ht
ml?url=WS27101fef35e777f9922804fbf5414e08-
8000.htm,topicNumber=d0e423159

c) above the Equator.


d) above the observer.
18. On the Universal Transverse Mercator projection
a) the origin of the longitude is located on
the west side of the projection area.
b) the origin of the latitude is at the
equator.
c) no false easting or northing applied
since origin is outside zone (northern
hemisphere).
d) scale factor of the central meridian is
1.00000.
18. On the Universal Transverse Mercator projection
a) the origin of the longitude is located on
the west side of the projection area.
b) the origin of the latitude is at the
equator.
c) no false easting or northing applied
since origin is outside zone (northern
hemisphere).
d) scale factor of the central meridian is
1.00000.
GEODESY PRINCIPLES
• UTM specifications
• Transverse Mercator projection with 6o wide zones
• Reference ellipsoid is GRS 80 in North America
• Origin of longitude is central meridian
• Origin of latitude is equator
• Unit of measure is meter
UTM Coordinate System

http://www.kwikplot.com/id7.html
GEODESY PRINCIPLES
• UTM specifications (cont.)
• False northing is 0 (for southern hemisphere, false
northing is 10,000,000 m
• False easting of 500,000m applied to central meridian of
each zone
• Scale factor of central meridian is 0.9996
• Zones numbered from 180o west (1) to 180o east (60)
• Extent in latitude is from 84o N to 80o S
19. A residual is the
a) difference between a measured
quantity and the most probable value.
b) mean of the squares of all the errors.
c) number of observations that are
greater than the number to solve for
the unknown values.
d) value for a measured quantity that has
the highest probability.
http://mathbits.com/MathBits/TISection/Statist
ics2/LeastSquares.htm

19. A residual is the


a) difference between a measured
quantity and the most probable value.
b) mean of the squares of all the errors.
c) number of observations that are
greater than the number to solve for
the unknown values.
d) value for a measured quantity that has
the highest probability.
ERRORS IN MEASUREMENTS
• The most probable value is the value for a measured quantity that has
the highest probability (d)
• Generally the arithmetic mean

• Degrees of freedom is the number of observations greater than the


number to solve for the unknown values
• Equals the number of redundant observations
ERRORS IN MEASUREMENTS
• Variance is the mean square of all the errors
• Value is used to express the precision of a group of measurements

• True value is the theoretically correct or exact value of a quantity


20. The range determined from a GPS measurement is
the wavelength times the
a) integer number of full wavelengths between the
satellite and receiver.
b) integer number of full wavelengths between the
satellite and receiver divided by 2.
c) combined fractional phase and integer number
of full wavelengths.
d) combined fractional phase and integer number
of full wavelengths divided by 2.
20. The range determined from a GPS measurement is
the wavelength times the
a) integer number of full wavelengths between the
satellite and receiver.
b) integer number of full wavelengths between the
satellite and receiver divided by 2.
c) combined fractional phase and integer number
of full wavelengths.
d) combined fractional phase and integer number
of full wavelengths divided by 2.
RECOMMENDED REFERENCES
• Elementary Surveying: An Introduction to
Geomatics, 12th edition by C. Ghilani and P. Wolf.
• Surveying: Theory and Practice, 7th edition by J.
Anderson and E. Mikhail.
• Adjustment Computations: Statistics and Least
Squares in Surveying and GIS, 2nd edition by C.
Ghilani and P. Wolf.
• The Global Positioning System and GIS, 3rd
edition by M. Kennedy.
• GPS Satellite Surveying, 3rd edition by A. Leick.
Photogrammetry
Preparation for ASPRS Certification: General Knowledge Workshop
WHAT IS PHOTOGRAMMETRY?

https://image.slidesharecdn.com/lectureonphotogrammetry-130817093510-phpapp01/95/lecture-
• Art, science, and technology of obtaining reliable
information about physical objects and the environment
through processes of recording, measuring, and
interpreting photographic images and patterns of
recorded radiant electromagnetic energy and other

on-photogrammetry-10-638.jpg?cb=1376733509
phenomenon
• Generally implies using photographs
• Growing importance of digital imagery
1. A linear array of sensors mounted in the focal plane of a single
lens camera
a) captures a full 2-dimensional array simultaneously.
b) produces a single row of the image at a time
perpendicular to the flight direction.
c) scans a single row of the image in the direction
parallel to the direction of flight.
d) uses a scanning mirror perpendicular to the flight
direction.
1. A linear array of sensors mounted in the focal plane of a single
lens camera
a) captures a full 2-dimensional array simultaneously.
b) produces a single row of the image at a time
perpendicular to the flight direction.
c) scans a single row of the image in the direction
parallel to the direction of flight.
d) uses a scanning mirror perpendicular to the flight
direction.
From: https://www.e-education.psu.edu/geog480/node/494

Cameras
• The linear array in the sensor is oriented
perpendicular to the flight direction
• Oftentimes referred to as pushbroom
scanner

• Answer a) pertains to a full-scan array

• Answer d) is actually a mirror scanning


system
• Referred to as a whiskbroom scanner.
2. A low oblique aerial photo is one in which the camera was
a) intentionally tilted containing the horizon.
b) intentionally tilted and not containing the horizon.
c) unintentionally tilted and containing the horizon.
d) unintentionally tilted and not containing the
horizon.
2. A low oblique aerial photo is one in which the camera was
a) intentionally tilted containing the horizon.
b) intentionally tilted and not containing the horizon.
c) unintentionally tilted and containing the horizon.
d) unintentionally tilted and not containing the
horizon.
OBLIQUE PHOTOGRAPHY
• Camera intentionally
tilted
• High oblique shows
the horizon
• Low oblique does not
contain the horizon
3. The refractive index relates to the
a) alignment of the lens elements within the lens cone.
b) changing of the direction of light when it hits a
mirror.
c) ratio of the absorbed to scattered energy within the
atmosphere.
d) speed of light as it travels through a medium.
3. The refractive index relates to the
a) alignment of the lens elements within the lens cone.
b) changing of the direction of light when it hits a
mirror.
c) ratio of the absorbed to scattered energy within the
atmosphere.
d) speed of light as it travels through a medium.
PHYSICS OF PHOTOGRAMMETRY
• Refractive Index
• As light passes through
different media with different
refractive indices, it gets bent
• Example is sticking an arm in
a lake

c speed in vacuum
ref index = =
v speed in medium
PHYSICS OF PHOTOGRAMMETRY
• Answer b actually deals • c relates to what happens
with reflection to energy as it passes
• Light ray is reflected off through the atmosphere.
the surface • Absorption
• Scattering
• Angle of incidence equals
the angle of reflection
From:
http://micro.magnet.fsu.edu/optics/l
ightandcolor/refraction.html
4. If the image distance (i) is equal to the focal length (f),
what is the object distance (o)?
a) o = i
b) o= i2 +f 2
c) o = 1
d) o = infinity
4. If the image distance (i) is equal to the focal length (f),
what is the object distance (o)?
a) o = i
b) o= i2 +f 2
c) o = 1
d) o = infinity
PHYSICS OF PHOTOGRAMMETRY
• From the lens equation,

1 1 1
+ =
o i f
From:
http://www.bibmath.net/bios/inde
x.php?action=affiche&quoi=hospital

it is clear that if o = ∞ then i = f


(L'Hôpital's rule) From http://education-
portal.com/academy/lesson/wha
t-is-lhopitals-rule.html#lesson
5. The intersection of a line from the perspective center to the
focal plane when centering a camera within a camera calibration
collimator system is the _____________.
a) calibrated principal point
b) indicated principal point
c) principal point of autocollimation
d) principal point of photogrammetry
5. The intersection of a line from the perspective center to the
focal plane when centering a camera within a camera calibration
collimator system is the _____________.
a) calibrated principal point
b) indicated principal point
c) principal point of autocollimation
d) principal point of photogrammetry
Cameras
• Before the lens is inserted in
place, an optical collimator
is pointed at the object
space target
• Camera place on line and
centered using the
collimator
• Because of lens, light rays
are bent
Cameras

a) calibrated principal point –


location of the principal point
from the camera calibration
adjustment
b) indicated principal point –
intersection of lines from
opposite fiducial marks.

• Additionally, within optics one can


find the principal point of
geometric optics – relates to the
physics of lenses
USGS Optical Science Lab (OSL) was
decommissioned in September of 2017 and no
longer certifies cameras for use in USGS flight
contracts.
6. For most photographic processes, a subtractive color process is
used. The subtraction of green from white yields
_____________.
a) cyan
b) magenta
c) red
d) yellow
6. For most photographic processes, a subtractive color process is
used. The subtraction of green from white yields
_____________.
Cyan – white – red
a) cyan
Red – primary color
b) magenta Yellow – white – blue
c) red
d) yellow While photography uses
subtractive colors, displays on
computer monitor or TV uses
additive colors
7. The intervalometer interval depends on _____________.
a) a, b, e a) aircraft velocity
b) a, c, f b)
c)
crab angle
diaphragm opening
c) a, d, e d) flying height
e) focal length
d) c, d, f f) shutter speed
7. The intervalometer interval depends on _____________.
a) a, b, e a) aircraft velocity
b) a, c, f b)
c)
crab angle
diaphragm opening
c) a, d, e d) flying height
e) focal length
d) c, d, f f) shutter speed
8. Moving the film slightly across the focal plane during the
exposure to compensate for image blur is referred to as
_____________.
a) exposure control
b) forward motion compensation
c) Intervalometer
d) shutter speed
8. Moving the film slightly across the focal plane during the
exposure to compensate for image blur is referred to as
_____________.
a) exposure control
b) forward motion compensation
c) intervalometer
d) shutter speed
http://rscc.umn.edu/rscc/v1m3images/cameradiag.gif
Pacey, R. and P. Fricker, 2005. “Forward Motion Compensation (FMC) – Is It the Same in the Digital Imaging World”, PERS, November.

Forward Motion Compensation


• 3 ways to limit effects of aircraft’s forward motion
• Using exposure time shorter or equal to pixel size
• Mechanically moving the focal plane
• Applying TDI (Time Delayed Integration)
• TDI
• Specialized CCD detector readout mode
• At lower altitudes where fast shutter speed required to limit image smear, image on
detector is read continuously, one row of pixels at a time from bottom of the
detector chip
• As each row is read, signals in the remaining detector pixels are shifted down by one
row causing the latent image to translate down the detector
• Therefore image smear in one captured image pixel with TDI is never more than that
of the Ground Sampling Distance (GSD)
9. Assume a digital image has DN values in the range of 15 to 65.
How would you characterize this image?
a) Good contrast
b) Poor contrast
c) Contrast will depend on the ground control
d) Contrast will depend on the sensors
Probably the “best” answer, given the options, is b) Poor
contrast, but it is not necessary the correct answer.

9. Assume a digital image has DN values in the range of 15 to 65.


How would you characterize this image?
a) Good contrast
b) Poor contrast
c) Contrast will depend on the ground control
d) Contrast will depend on the sensors
Ordinal measures are poor discriminators for
test questions. Good, poor can vary from
instance to instance
10. A distance between two street intersections measured on a
vertical photograph was found to be 4.52”. The ground distance
is known to be 1,325.0 feet. The scale of the photograph,
expressed as a representative fraction, is _______.
a)1 : 24
b)1 : 293
c) 1 : 3,518
d)1 : 5,989
10. A distance between two street intersections measured on a
vertical photograph was found to be 4.52”. The ground distance
is known to be 1,325.0 feet. The scale of the photograph,
expressed as a representative fraction, is _______.
a)1 : 24
b)1 : 293
c) 1 : 3,518
d)1 : 5,989
Scale
ab 4.52" 1"
S= = =
AB 1,325.0′ 293.142′

To convert to representative fraction:

1" 1′ 1
𝑆𝑆 = =
293.142′ 12" 3,518
11. A photograph with a scale of 1” = 660’ is scanned at a resolution
of 1,700 dpi. What is the corresponding ground sample
distance?
a) 0.39 feet/pixel
b) 0.88 feet/pixel
c) 1.29 feet/pixel
d) 2.58 feet/pixel
11. A photograph with a scale of 1” = 660’ is scanned at a resolution
of 1,700 dpi. What is the corresponding ground sample
distance?
a) 0.39 feet/pixel
b) 0.88 feet/pixel ab in�dot
S= =
c) 1.29 feet/pixel AB
𝑖𝑖𝑖𝑖�
GSD
1"� 1" 660′
d) 2.58 feet/pixel 𝐺𝐺𝐺𝐺𝐺𝐺 = 𝑑𝑑𝑑𝑑𝑑𝑑 = 1700 = = 0.39′
𝑆𝑆 1"�660′ 1700 1"
12. An omega adjustment
a) rotates the model about the x-axis.
b) rotates the model about the y-axis.
c) rotates the model about the z-axis.
d) translates the model up and down.
12. An omega adjustment
a) rotates the model about the x-axis.
b) rotates the model about the y-axis.
c) rotates the model about the z-axis.
d) translates the model up and down.
Orientation
• Models require 5 movement
• 3 Rotations:
• Kappa about the z-axis
• Phi about the y-axis
• Omega about the x-axis
• 2 Translations
• by – displaces image in y-direction
• bz – displaces image in z-direction
• Remaining element is bx – used for scale
13. In photogrammetry, the minimum number of parallax
measurements for relative orientation is ______.
a) 4
b) 5
c) 6
d) 9
 2

- py =
XY
(dφ - dφ") -  Z +  (dω - dω") - (d′bz - dbz" - b • dφ")
′ Y ′
Y
Z  Z Z

+ X (dκ′ - dκ") + (d′by - dby + d • dκ")


13. In photogrammetry, the minimum number of parallax
measurements for relative orientation is ______.
a) 4 p1 o1
o 2 p2
r2

b) 5 r1 X
b
bx by
bz

c) 6 Y Z
R2

d) 9
R1

H YG P

XG
Photogrammetry Principles
• The minimum number is 5
• There are 5 types of parallaxes and this give a
unique solution
• Generally the 6 von Gruber points are used
• Gives a check on the parallax measurements
• 9 is the recommended method when using
analytical/digital plotters
• Use 6 von Gruber points and points between
14. Georeferencing is
a) relating features to the ground coordinate system.
b) relating features to the sensor orientation
c) relating features to each other.
d) relating features to digital imagery.
14. Georeferencing is
a) relating features to the ground coordinate system.
b) relating features to the sensor orientation
c) relating features to each other.
d) relating features to digital imagery.
15. Using the two-dimensional Helmert (conformal) transformation
requires a point with coordinates in both systems. The
minimum number of points is _____.
a) 2
b) 3
c) 4
d) 5
15. Using the two-dimensional Helmert (conformal) transformation
requires a point with coordinates in both systems. The
minimum number of points is _____.
a) 2
b) 3
c) 4
d) 5
Affine Transformation
• The Conformal, or isogonal affine, transformation
requires 2 points
x ' = x ⋅ s ⋅ cos α + y ⋅ s ⋅ sin α + ∆x
y' = − x ⋅ s ⋅ sin α + y ⋅ s ⋅ cos α + ∆y
• 4 unknowns: s, α, ∆x, ∆y therefore need 4
observations
16. The probable error indicates that
a) 50% of all errors will occur within the limits.
b) 57.51% of all errors will occur within the limits.
c) 68.27% of all errors will occur within the limits.
d) 90% of all errors will occur within the limits.
e) 99.73% of all errors will occur within the limits.
16. The probable error indicates that
a) 50% of all errors will occur within the limits.
b) 57.51% of all errors will occur within the limits.
c) 68.27% of all errors will occur within the limits.
d) 90% of all errors will occur within the limits.
e) 99.73% of all errors will occur within the limits.
Errors in Measurements
n
• 57.51% -- Called the average error
• Defined as
∑x i −x
AE = i =1
n
• 68.27% -- Called the standard error

∑ (x )
• Defined as n
2
i −x
σ= i =1

n −1
ERRORS IN MEASUREMENTS
• 90% -- Called the 90% error
• Basis for the National Map Accuracy Standards

• 99.73% - Called the 3 sigma (3σ) error


• 3 times the standard error
• Often used as criteria for blunder detection
ERRORS IN MEASUREMENTS
Error Probability Conversion Factor
wrt σ
Probable Error 0.5000 0.6745 σ
Average Error 0.5751 0.7979 σ
Standard Error 0.6827 1.0000 σ
90% Error 0.9000 1.6449 σ
3σ Error 0.9973 3.0000 σ
ASPRS Accuracy Standards
• ASPRS Positional Accuracy
Standards for Digital Geospatial
• Approved by the ASPRS Board of
Directors on November 17, 2014
• Supersedes and replaces prior
ASPRS accuracy standards, including
the ASPRS Accuracy Standards for
Large Scale Maps (1990) and ASPRS
Vertical Accuracy Reporting for
Lidar Data (2004).
17. Conformance requirements for the ASPRS Accuracy Standards
for Digital Geospatial Data are expressed by ______________.
a) map classification (Class 1, Class 2, etc.)
b) map scale conformance
c) survey class accuracies (First-order, Second order, etc.)
d) none of the above
http://www.asprs.org/a/society/committees/standards/ASPRS_Positional_Accuracy_Sta
ndards_Edition1_Version100_November2014.pdf

17. Conformance requirements for the ASPRS Accuracy Standards


for Digital Geospatial Data are expressed by ______________.
a) map classification (Class 1, Class 2, etc.)
b) map scale conformance
c) survey class accuracies (First-order, Second order, etc.)
d) none of the above

There are no conformance requirements


established for this standard.
18. Vertical accuracy, according to the ASPRS Accuracy Standard for
Digital Geospatial Data, is to be assed using ________ statistics
in non-vegetated terrain and _______ statistics in vegetated
terrain.
a) RMSEZ; RMSEZ
b) RMSEZ; 95th percentile
c) 95th percentile; RMSEZ
d) 95th percentile; 95th percentile
http://www.asprs.org/a/society/committees/standards/ASPRS_Positional_Accuracy_Sta

18. Vertical accuracy, according to the ASPRS Accuracy Standard for


ndards_Edition1_Version100_November2014.pdf

Digital Geospatial Data, is to be assed using ________ statistics


in non-vegetated terrain and _______ statistics in vegetated
terrain.
a) RMSEZ; RMSEZ
b) RMSEZ; 95th percentile
c) 95th percentile; RMSEZ
d) 95th percentile; 95th percentile
http://www.asprs.org/a/society/committees/standards/ASPRS_Positional_Accuracy_Sta
http://www.asprs.org/a/society/committees/standards/ASPRS_Positional_Accuracy_Sta
ndards_Edition1_Version100_November2014.pdf
ndards_Edition1_Version100_November2014.pdf

ASPRS Accuracy Standards


• Non-vegetated terrain – errors typically follow a
normal distribution suitable for RMSE statistical
analysis
• Vegetated terrain – errors do not necessarily follow a
normal distribution. Here the 95th percentile value
more fairly estimates accuracy at the 95% confidence
level
http://www.asprs.org/a/society/divisions/pad/Accuracy/ASPRS%20Accuracy%20Standards_PECORA2014_Slides.pptx

New Standard Highlights


• It is All Metric!
• Unlimited Horizontal Accuracy Classes:
Horizontal Accuracy Standards for
Geospatial Data
Horizontal Accuracy at
Horizontal Orthoimagery Mosaic
RMSEx and 95% Confidence Level
Accuracy Class RMSEr (cm) Seamline Mismatch
RMSEy (cm) (cm)
(cm)

X-cm ≤X ≤1.41*X ≤2.45*X ≤ 2*X


Horizontal Accuracy Class RMSEr Orthoimage Mosaic Seamline Horizontal Accuracy at the 95%
RMSEx and RMSEy (cm) (cm) Maximum Mismatch (cm) Confidence Level (cm)
http://www.asprs.org/a/society/divisions/pad/Accuracy/ASPRS%20Accuracy%20Standards_PE
CORA2014_Slides.pptx
0.63 0.9 1.3 1.5
1.25 1.8 2.5 3.1
2.50 3.5 5.0 6.1
5.00 7.1 10.0 12.2

Common Horizontal 7.50


10.00
10.6
14.1
15.0
20.0
18.4
24.5
Accuracy Classes 12.50 17.7 25.0 30.6
15.00 21.2 30.0 36.7
according to the 17.50 24.7 35.0 42.8

new standard 20.00


22.50
28.3
31.8
40.0
45.0
49.0
55.1
25.00 35.4 50.0 61.2
27.50 38.9 55.0 67.3
30.00 42.4 60.0 73.4
45.00 63.6 90.0 110.1
60.00 84.9 120.0 146.9
75.00 106.1 150.0 183.6
100.00 141.4 200.0 244.8
150.00 212.1 300.0 367.2
200.00 282.8 400.0 489.5
250.00 353.6 500.0 611.9
300.00 424.3 600.0 734.3
500.00 707.1 1000.0 1223.9
1000.00 1414.2 2000.0 2447.7
http://www.asprs.org/a/society/divisions/pad/Accuracy/ASPRS%20Accuracy%20Standards_PE
CORA2014_Slides.pptx
Associated Horizontal Accuracy
According to Legacy ASPRS 1990
Standard
Common
Associated Map ASPRS 1990
Orthoimagery Pixel
Scale Accuracy Class
Sizes
RMSEx and RMSEy RMSEx and RMSEy
(cm) in terms of pixels

Examples on Horizontal Accuracy 1 1.3 2-pixels

for Digital Orthoimagery 0.625 cm 1:50 2

3
2.5

3.8
4-pixels

6-pixels

interpreted from ASPRS 1990 1 2.5 2-pixels

Legacy Standard. 1.25 cm 1:100 2

3
5.0

7.5
4-pixels

6-pixels

1 5.0 2-pixels

2.5 cm 1:200 2 10.0 4-pixels

3 15.0 6-pixels

1 10.0 2-pixels

5 cm 1:400 2 20.0 4-pixels

3 30.0 6-pixels

1 15.0 2-pixels

7.5 cm 1:600 2 30.0 4-pixels

3 45.0 6-pixels

1 30.0 2-pixels

15 cm 1:1,200 2 60.0 4-pixels

3 90.0 6-pixels
Recommended
Horizontal Accuracy
Common Class RMSEx and Orthoimage RMSEx
Orthoimagery Pixel RMSE y (cm) and RMSEy in terms of Recommended use
Sizes pixels

≤1.3 ≤1-pixel Highest accuracy work

Digital Orthoimagery Accuracy 1.25 cm


2.5 2-pixels
Standard Mapping and GIS
work

Examples for Current Large and ≥3.8 ≥3-pixels


Visualization and less
accurate work

Medium Format Metric Cameras ≤2.5 ≤1-pixel Highest accuracy work


Standard Mapping and GIS
5.0 2-pixels
2.5 cm work
Visualization and less
≥7.5 ≥3-pixels
accurate work
≤5.0 ≤1-pixel Highest accuracy work
Standard Mapping and GIS
10.0 2-pixels
5 cm work
Visualization and less
≥15.0 ≥3-pixels
accurate work
≤7.5 ≤1-pixel Highest accuracy work
Standard Mapping and GIS
http://www.asprs.org/a/society/divisions/pad/Accuracy/ASPRS%20Accuracy%20Standards_PECO 15.0 2-pixels
RA2014_Slides.pptx 7.5 cm work
Visualization and less
≥22.5 ≥3-pixels
accurate work
≤15.0 ≤1-pixel Highest accuracy work
Standard Mapping and GIS
30.0 2-pixels
15 cm work
Visualization and less
≥45.0 ≥3-pixels
accurate work
Horizontal Accuracy/Quality Examples for High Accuracy Digital
Planimetric Data
Equivalent to map scale
ASPRS 2014
in
Horizontal
Horizontal Accuracy ASPRS ASPRS
RMSEr Accuracy at the Approximate GSD of
Class RMSEx and 1990 Class 1990 Class Equivalent to map
RMSEy (cm)
(cm) 95% Confidence Source Imagery (cm)
1 2 scale in NMAS
Level (cm)
0.63 0.9 1.5 0.31 to 0.63 1:25 1:12.5 1:16
1.25 1.8 3.1 0.63 to 1.25 1:50 1:25 1:32
2.5 3.5 6.1 1.25 to 2.5 1:100 1:50 1:63
5.0 7.1 12.2 2.5 to 5.0 1:200 1:100 1:127
7.5 10.6 18.4 3.8 to 7.5 1:300 1:150 1:190
10.0 14.1 24.5 5.0 to 10.0 1:400 1:200 1:253
12.5 17.7 30.6 6.3 to12.5 1:500 1:250 1:317
15.0 21.2 36.7 7.5 to 15.0 1:600 1:300 1:380
17.5 24.7 42.8 8.8 to 17.5 1:700 1:350 1:444
20.0 28.3 49.0 10.0 to 20.0 1:800 1:400 1:507
22.5 31.8 55.1 11.3 to 22.5 1:900 1:450 1:570
http://www.asprs.org/a/socie 25.0 35.4 61.2 12.5 to 25.0 1:1000 1:500 1:634
ty/divisions/pad/Accuracy/AS 27.5 38.9 67.3 13.8 to 27.5 1:1100 1:550 1:697
PRS%20Accuracy%20Standard
s_PECORA2014_Slides.pptx 30.0 42.4 73.4 15.0 to 30.0 1:1200 1:600 1:760
http://www.asprs.org/a/society/divisions/pad/Accuracy/ASPRS%20Accuracy%20Standards_PE
CORA2014_Slides.pptx

New Standard Highlights


• Unlimited Vertical Accuracy Classes:
Vertical Accuracy Standards for
Digital Elevation Data
Absolute Accuracy Relative Accuracy (where applicable)
VVA Within- Swath Swath-to-Swath Swath-to-Swath
Vertical RMSEz NVA at 95% at 95th Hard Surface Non-Vegetated Non-Vegetated
Accuracy Class Non-Vegetated Confidence Level Perce Repeatability Terrain Terrain
(cm) (cm) ntile (Max Diff) (RMSDz) (Max Diff)
(cm) (cm) (cm) (cm)
≤3.00
X-cm ≤X ≤1.96*X ≤0.60*X ≤0.80*X ≤1.60*X
*X
NATIONAL STANDARD FOR SPATIAL DATA
ACCURACY

• Uses rms error like


ASPRS
• Accuracy reported at the
95% confidence level
• No threshold specified
• Up to the contracting
agency
• Can be related to NMAS
19. Ground control accuracy (RMSE) for planimetric data production
shall be __________.
a) equal to the RMSEMap
b) 1/2 * RMSEMap
c) 1/3 * RMSEMap
d) 1/4 * RMSEMap
From: New Standards for New Era: Overview of the 2015 ASPRS Positional
Accuracy Standards for Digital Geospatial Data, PERS, March 2015.

19. Ground control accuracy (RMSE) for planimetric data production


shall be __________.
a) equal to the RMSEMap
b) 1/2 * RMSEMap
c) 1/3 * RMSEMap
d) 1/4 * RMSEMap
20. The raw uncorrected image obtained from a linear array camera
is ______________________________.
a) curved concave to the earth’s rotation
b) offset in step fashion along the flight direction
c) wavy parallel to the flight direction
d) wavy perpendicular to the flight direction
Paul Wolf, Bon Dewitt, Benjamin Wilkinson, 2014. Elements of Photogrammetry with
Applications in GIS, 4th edition.

20. The raw uncorrected image obtained from a linear array camera
is ______________________________.
a) curved concave to the earth’s rotation
b) offset in step fashion along the flight direction
c) wavy parallel to the flight direction
d) wavy perpendicular to the flight direction
Paul Wolf, Bon Dewitt, Benjamin Wilkinson, 2014. Elements of Photogrammetry with
Applications in GIS, 4th edition.

• Uncorrected Image • Corrected Image


21. A digital camera has an imaging array of 10,000 x 15,000 pixels
(150 megapixels). If the individual CCD element is 7 µm, how
big of an image can this camera capture?
a) 36.3 x 54.4 mm
b) 70 x 105 mm
c) 142 x 214 mm
d) 700 x 1050 mm
Paul Wolf, Bon Dewitt, Benjamin Wilkinson, 2014. Elements of Photogrammetry with
Applications in GIS, 4th edition.

21. A digital camera has an imaging array of 10,000 x 15,000 pixels


(150 megapixels). If the individual CCD element is 7 µm, how
big of an image can this camera capture?
a) 36.3 x 54.4 mm
b) 70 x 105 mm
c) 142 x 214 mm 0.007 𝑚𝑚𝑚𝑚/𝑝𝑝𝑝𝑝𝑝𝑝𝑝𝑝𝑝𝑝
= 70 𝑚𝑚𝑚𝑚
d) 700 x 1050 mm 10,000 𝑝𝑝𝑝𝑝𝑝𝑝𝑝𝑝𝑝𝑝𝑝𝑝
0.007 𝑚𝑚𝑚𝑚/𝑝𝑝𝑝𝑝𝑝𝑝𝑝𝑝𝑝𝑝
= 105 𝑚𝑚𝑚𝑚
15,000 𝑝𝑝𝑝𝑝𝑝𝑝𝑝𝑝𝑝𝑝𝑝𝑝
22. The ASPRS Positional Accuracy Standards defines the use of
_______________ as the equivalent to dashed contours utilized
in past standards.
a) low confidence area
b) non-vegetated vertical accuracy
c) occluded elevations
d) vegetated vertical accuracy
From: New Standards for New Era: Overview of the 2015 ASPRS Positional
Accuracy Standards for Digital Geospatial Data, PERS, March 2015.

22. The ASPRS Positional Accuracy Standards defines the use of


_______________ as the equivalent to dashed contours utilized
in past standards.
a) low confidence area
b) non-vegetated vertical accuracy
c) occluded elevations
d) vegetated vertical accuracy
Low Confidence Areas for Elevation Data

From: New Standards for New Era: Overview of the 2015 ASPRS Positional Accuracy Standards for Digital Geospatial Data, PERS, March 2015.
23. What is the minimum flying height for a fixed-wing aircraft over
a town?
a) 500’ above the highest obstacle
b) 1,000’ above the highest obstacle
c) 1,500’ above the highest obstacle
d) Anywhere
From: https://www.law.cornell.edu/cfr/text/14/91.119

23. What is the minimum flying height for a fixed-wing aircraft over
a town?
a) 500’ above the highest obstacle
b) 1,000’ above the highest obstacle
c) 1,500’ above the highest obstacle
d) Anywhere

From:
http://www.vulcanair.com/photogallery/images/____3/lar
ge/___30.jpg
Minimum Flying Height

From: https://www.law.cornell.edu/cfr/text/14/91.119
24. The ratio of the flying height to the smallest contour interval
that can be accurately mapped is called the ___________.
a) base-height ratio
b) C-factor
c) crab ratio
d) neat model ratio
24. The ratio of the flying height to the smallest contour interval
that can be accurately mapped is called the ___________.
a) base-height ratio
b) C-factor
c) crab ratio
d) neat model ratio
http://www.adamtech.com.au/Blog/?p=245
Base : Height Ratio
72 : 120 = 1 : 1.67
Height Accuracy 1.67 times worse than planimetric
Photogrammetric Mapping
• The correct answer is C-factor
• Base-height ratio is ratio of air base to flying height of a stereo pair of
photographs
• Crab is not a ratio but an angle where the edges of the photography
are not parallel to air base
• Neat model is not a ratio but that part of the overlap area that is
utilized in mapping
• Generally a rectangle whose width equals the air base and whose length
equals the width between flight line
25. In airborne GPS, the location of the exposure station is
a) determined directly since the GPS sample rate will
trip the aerial camera shutter.
b) determined directly since the camera sends a signal
to the receiver to measure the GPS position.
c) determined directly by using the nearest GPS
measurement to the exposure time.
d) determined by interpolating the exposure time to
GPS sample measurements before and after the
exposure.
This question is an example of one where the answer has not kept up
with technology. Applanix does send a pulse to the camera to make
the exposure. Thus, a) is also a correct answer. Describe this at the
end of the exam in Comments.
25. In airborne GPS, the location of the exposure station is
a) determined directly since the GPS sample rate will
trip the aerial camera shutter.
b) determined directly since the camera sends a signal
to the receiver to measure the GPS position.
c) determined directly by using the nearest GPS
measurement to the exposure time.
d) determined by interpolating the exposure time to
GPS sample measurements before and after the
exposure.
Airborne GPS
 The correct answer is to interpolate the position of the exposure time (d)

 With a) and b) there is a time offset between when the signal is sent and
before the operation can be performed
 While e) is right, the advantage of airborne GPS is that the position of the
camera at the instant of exposure is known (measured)
RECOMMENDED REFERENCES
• Manual of Photogrammetry, 6th edition, ASPRS
• Digital Elevation Model Technologies and
Applications: The DEM Users Manual, 2nd edition,
ASPRS
• Elements of Photogrammetry, 4rd edition, by Paul
Wolf, Bon Dewitt and Benjamin Wilkinson
• Introduction to Modern Photogrammetry, by
Edward Mikhail, James Bethel, and Chris
McGlone
• Photogrammetry, 3rd edition, by Frank Moffit and
Edward Mikhail (out of print)
• Digital Photogrammetry, by Tony Schenk
https://mashable.com/2015/01/27/white-house-drone-
crash-drinking/#1e5.dSUWvqqV

Unmanned Autonomous
Systems
Preparation for ASPRS Certification: General Knowledge Workshop

http://gis.uww.edu/2016/06/suas-vegetation-
mapping-first-successful-data-collection/
What is an UAV/UAS
• Unmanned aircraft
• Fixed wing
• Rotor wing
• Light than air

• Unmanned a misnomer
• Humans still crucial to operate systems
Introduction to Unmanned Aircraft Systems by R. Barnhart, S. Hottman, D. Marshall and E. Shappee

History of Drones
• Chinese General Zhuge Liang (180-234 AD) used oil-burning lamps to
heat air in paper balloons
• Flew over enemy – Divine power at work
• Elmer Sperry, 1918, build drone for U.S. Navy
• Could be self-launched, fly unmanned for 1,000 yds. to target, detonate
warhead
• Germans mass-produced V-1 Buzz Bomb
• Cruise missile type aircraft, able to be launched from ground or air
• First jet-powered unmanned aircraft
• Delivered 1,800 lb.. warhead
• Primarily ground-launched rail system
1. The maximum weight for a UAS for work purposes is _____.
a) 10 lbs.
b) 25 lbs.
c) 55 lbs.
d) 80 lbs.
www.faa.gov/uas/getting_started

1. The maximum weight for a UAS for work purposes is _____.


a) 10 lbs.
b) 25 lbs.
c) 55 lbs.
d) 80 lbs.
2 . As a general rule, UAS commercial operations are restricted
to Class G airspace. Class G airspace is
a) airspace extending to 18,000 feet.
b) airspace between 1,200’ – 18,000’.
c) airspace extending to 2,500’.
d) airspace typically below 1,200’.
www.faa.gov/uas/getting_started

2 . As a general rule, UAS commercial operations are restricted


to Class G airspace. Class G airspace is
a) airspace extending to 18,000 feet.
b) airspace between 1,200’ – 18,000’.
c) airspace extending to 2,500’.
d) airspace typically below 1,200’.
https://en.wikipedia.org/wiki/Airspace_class_(United_States)#/media/File:Airspace_classes_(United_States).png
Airspace classification
Airspace classification

.
3 . No person may act as a remote pilot in command or visual
observer for more than ______ unmanned aircraft at one
time.
a) one
b) two
c) four
d) five
www.faa.gov/uas/getting_started

3 . No person may act as a remote pilot in command or visual


observer for more than ______ unmanned aircraft at one
time.
a) one
b) two
c) four
d) five
4 . The FAA requires a preflight inspection by ____________.
a) aircraft manufacturer
b) remote pilot in command
c) visual observer
d) any of the above
www.faa.gov/uas/getting_started

4 . The FAA requires a preflight inspection by ____________.


a) aircraft manufacturer
b) remote pilot in command
c) visual observer
d) any of the above
5. FAA airworthiness certification is required ___________.
a) prior to each flight
b) every 30 flights
c) yearly
d) certificates are not required
5. FAA airworthiness certification is required ___________.
a) prior to each flight
b) every 30 flights
c) yearly
d) certificates are not required
6. Which of the following is not a part of the Operating Rules
for flying a UAS for work or business (assume no waiver)?
a) Must fly at or below 100 mph.
b) Must fly during the day.
c) Must fly under 400’.
d) Must keep the aircraft within one mile of the remote
pilot in command.
6. Which of the following is not a part of the Operating Rules
for flying a UAS for work or business (assume no waiver)?
a) Must fly at or below 100 mph.
b) Must fly during the day.
c) Must fly under 400’.
d) Must keep the aircraft within one mile of the remote
pilot in command.
• Among the requirements to fly a UAS commercially include
flying below 400 feet, flying only during daytime and flying
less than 100 miles per hour. But the standout
requirement is that commercial drone operators will need
to take a written, in-person, drone-specific, aeronautical
knowledge test.
• http://www.drone360mag.com/news-notes/2016/06/faa-part-
107-uas-aeronautical-knowledge-test-everything-you-need-to-
know
• Daylight encompasses 30 minutes before local time official
sunrise and after local official sunset
• https://www.faa.gov/news/fact_sheets/news_story.cfm?newsId
=22615
7. A manual fixed-wing aircraft flying below the minimum
above ground level (AGL) required altitude enters the
airspace where a sUAS mission is being undertaken. Which
of the following is true?
a) Both flights can continue provided that visual contact is
maintained between aircraft.
b) The manned fixed-wing aircraft must abort.
c) The sUAS must abort.
d) Whoever enters the contested airspace has priority.
7. A manual fixed-wing aircraft flying below the minimum
above ground level (AGL) required altitude enters the
airspace where a sUAS mission is being undertaken. Which
of the following is true?
a) Both flights can continue provided that visual contact is
maintained between aircraft.
b) The manned fixed-wing aircraft must abort.
c) The sUAS must abort.
d) Whoever enters the contested airspace has priority.
8. According to FAA Section 333 Exemption, a NOTAM is
required for a sUAS flight.
a) True
b) False
https://www.faa.gov/uas/media/RIN_2120-AJ60_Clean_Signed.pdf

8. According to FAA Section 333 Exemption, a NOTAM is


required for a sUAS flight.
a) True
b) False Need to be careful researching on web.
Many sites state that NOTAM are required.
NOTAM (Notice to Airmen
The FAA also declines to impose additional NOTAM requirements on
small UAS operations. The NOTAM system is used to alert pilots of
conditions or situations in the NAS that could present a hazard to aircraft.
Historically, the FAA has used a NOTAM requirement in the COAs it issued
for UAS operations. This was appropriate because small UAS operations
were outside the regulatory structure that was then in place, and, while
not inherently hazardous, small UAS flights required exemption or waiver
from a number of FAA regulations. Because these operations deviated
from existing FAA regulations, a NOTAM was an acceptable means to
notify pilots of the activity. However, with part 107, the FAA is bringing a
subset of UAS operations within the FAA regulatory structure. Civil,
public, and military pilots are expected to be familiar with regulations
affecting their flight, including the possibility of encountering UAS activity
below 400 feet. Therefore, requiring a NOTAM would not be appropriate.
9. The difference between traditional softcopy
photogrammetry and structure-from-motion (SfM)
photogrammetry is that
a) SfM does not require 3-D pose of the camera or a series
of ground control points.
b) SfM is based on lidar or radar range measurements.
c) traditional photogrammetry does not require as much
control in the triangulation.
d) traditional photogrammetry requires stereo viewing
from 2 points in space where SfM does not.
M. Westoby et al, 2012. “’Structure-from-Motion Photogrammetry: A Low-Cost, Effective
Tool for Geoscience Applications”, Geomorphology, 179.

9. The difference between traditional softcopy


photogrammetry and structure-from-motion (SfM)
photogrammetry is that
a) SfM does not require 3-D pose of the camera or a series
of ground control points.
b) SfM is based on lidar or radar range measurements.
c) traditional photogrammetry does not require as much
control in the triangulation.
d) traditional photogrammetry requires stereo viewing
from 2 points in space where SfM does not.
10. The requirement for commercial drone operations is that
they need to take a(n) _____________ aeronautical
knowledge test.
a) computer generated
b) interview based
c) drone specific
d) web based
https://www.geospatialworld.net/news/free-precisionmapper-encourage-innovation-
10. The requirement for commercial drone operations is that
they need to take a(n) _____________ aeronautical
knowledge test.
a) computer generated
b) interview based
c) drone specific
d) web based

drone-mapping-become-easy/
How do I become a certified drone pilot?
• Under Part 107, which the FAA announced on June 21st, 2016 and
implemented on August 29th, 2016, commercial drone operators
are required to:
• Pass an initial aeronautical knowledge test at one of 690 FAA-approved
knowledge testing centers across the United States (this list last updated
July 2016). That’s what our Part 107 training course prepares you for.
• Be vetted by the Transportation Security Administration.
• Obtain a Remote Pilot Certificate with a small UAS rating (like existing pilot
airman certificates, never expires).
• Pass a recurrent aeronautical knowledge test every 24 months.
• Be at least 16 years old.
• Make available to the FAA, upon request, the small UAS for inspection or
testing, and any associated documents/records required to be kept under
the proposed rule.
• Report an accident to the FAA within 10 days of any operation that results in
injury or property damage over $500.
11. The FAA Airspace Authorization Grid Map for Class E
Airspace are issued to assist drone operators. Which of the
following statements are true.
a) A value of “0” on a grid cell identifies areas where no
authorization is required.
b) Identifies areas near airports were FAA authorization is
not required.
c) Identifies areas where FAA safety analysis has been pre-
approved.
d) Maps are only informational and do not grant operators
permission to fly sUASs.
11. The FAA Airspace Authorization Grid Map for Class E
Airspace are issued to assist drone operators. Which of the
following statements are true.
a) A value of “0” on a grid cell identifies areas where no
authorization is required.
b) Identifies areas near airports were FAA authorization is
not required.
c) Identifies areas where FAA safety analysis has been pre-
approved.
d) Maps are only informational and do not grant operators
permission to fly sUASs.
FAA Airspace Authorization Grid Map for
Class E Airspace
• The maps depict areas and altitudes near airports where UAS may operate safely. But drone operators still need FAA authorization to
fly in those areas.
• The maps are informational and do not give people permission to fly drones. Remote pilots must still submit an online airspace
authorization application.
• The map viewer displays numbers in grid cells which represent the distances Above Ground Level (AGL) in one square mile up to 400
feet where drones may fly. Zeros indicate critical locations around airports and other aircraft operating areas, like hospital helipads,
where no drone flights can be preauthorized.
• Requests to operate in these areas will require further coordination and FAA safety analysis, which can result in additional safety
mitigations to be complied with by the drone operator.
• Remote pilots can refer to the maps to tailor their requests to align with locations and altitudes when they complete airspace
authorization applications.
• Altitudes that exceed those depicted on the maps require additional safety analysis and coordination to determine if an application
can be approved.
• Additional maps will be published every 56 days through the end of the year. The updates will coincide with the agency’s existing 56-
day aeronautical chart production schedule (PDF). If a map is not yet available, it can be expected in future releases.
• The next grid maps should be released on: 25-May, 22-Jun, 20-Jul
• The digital maps being made public through the FAA UAS Data Delivery System include a grid overlay that depicts acceptable altitude
limits for UAS operations near airports around the country. Remote pilots should use these maps to plan drone operations and
drafting their submissions for airspace authorization requests. We presume that requests that fall within the grid map parameters
on location and altitude will be more likely to achieve rapid approval by the FAA.
https://www.dronelaw.pro/faa-airspace-authorization-grid-maps-class-e-airspace/
FAA Airspace Authorization Grid Map for Class
E Airspace https://www.dronelaw.pro/faa-airspace-authorization-grid-maps-class-e-airspace/
FAA Airspace Authorization Grid Map for
Class E Airspace

https://www.dronelaw.pro/faa-airspace-
authorization-grid-maps-class-e-airspace/
12. You are flying over a project area where the visual observer
notices what could be a figure sunbathing near the project
area. He/She should
a) Fly over the person’s property as quietly as possible.
b) Gather the data because it may be needed in the future.
c) Not invade the individual’s privacy by taking pictures.
d) Secure any pictures you might have taken because of it’s
sensitive nature.
12. You are flying over a project area where the visual observer
notices what could be a figure sunbathing near the project
area. He/She should
a) Fly over the person’s property as quietly as possible.
b) Gather the data because it may be needed in the future.
c) Not invade the individual’s privacy by taking pictures.
d) Secure any pictures you might have taken because of it’s
sensitive nature.
Guidelines for Neighborly Drone Use:
• If you can, tell other people you’ll be taking pictures or video of them before you do so.
• If you think someone has a reasonable expectation of privacy, don’t violate that privacy by
taking pictures, video, or otherwise gathering sensitive data, unless you’ve got a very good
reason.
• Don’t fly over other people’s private property without permission if you can easily avoid doing
so.
• Don’t gather personal data for no reason, and don’t keep it for longer than you think you have
to.
• If you keep sensitive data about other people, secure it against loss or theft.
• If someone asks you to delete personal data about him or her that you’ve gathered, do so,
unless you’ve got a good reason not to.
• If anyone raises privacy, security, or safety concerns with you, try and listen to what they have
to say, as long as they’re polite and reasonable about it.
• Don’t harass people with your drone.
Vetoed
13. What is the minimum weather visibility from the UAS
control station?
a) 1 mile
b) 2 miles
c) 3 miles
d) 5 miles
https://www.faa.gov/news/fact_sheets/news_story.cfm?newsId=22615

13. What is the minimum weather visibility from the UAS


control station?
a) 1 mile
b) 2 miles
c) 3 miles
d) 5 miles

https://dronelife.com/wp-content/uploads/2018/01/Yellowscan_LIDAR_on_OnyxStar_FOX-
C8_HD-e1514909230297.jpg
14. UASs weighing ____________________ are required to be
registered with the FAA.
a) 0.55 lbs. to just less than 55 lbs.
b) 1 lb. to just less than 55 lbs.
c) 2.55 lbs. to just less than 55 lbs.
d) 5 lbs. to just less than 55 lbs.
https://federaldroneregistration.com/faq/

14. UASs weighing ____________________ are required to be


registered with the FAA.
a) 0.55 lbs. to just less than 55 lbs.
b) 1 lb. to just less than 55 lbs.
c) 2.55 lbs. to just less than 55 lbs.
d) 5 lbs. to just less than 55 lbs.

https://faadronezone.faa.gov/#/
15. What is the minimum age requirement for registering a
sUAS?
a) 10 years old
b) 13 years old
c) 15 years old
d) 18 years old
e) 21 years old
https://federaldroneregistration.com/faq/

15. What is the minimum age requirement for registering a


sUAS?
a) 10 years old
b) 13 years old
c) 15 years old
d) 18 years old
e) 21 years old
16. If an individual already has a current pilot certificate under
FAA Part 61 then they do not need a remote pilot certificate.
a) True
b) False
16. If an individual already has a current pilot certificate under
FAA Part 61 then they do not need a remote pilot certificate.
a) True
b) False
From the FAA
(https://faa.custhelp.com/app/answers/detail/a_id/748/kw/Aeronautical%20Knowledge%20Testing%20and%20Remote%20Pilot%20Certif
ication%20Questions):
To act as a remote pilot in command under Part 107, you must have a remote pilot certificate.
However, part 61 pilot certificate holders who have completed a flight review within the past 24
months may elect to take an online training course focusing on UAS-specific areas
of knowledge instead of the aeronautical knowledge test. The online training for
current pilot certificate holders is available at www.faasafety.gov (ALC-451: Part 107 Small
Unmanned Aircraft Systems). All other members of the public must take and pass the
initial aeronautical knowledge test to obtain a remote pilot certificate.
17. Using a DSLR with a focal length of 35mm on a sUAS,
calculate the flying height if the pixel size is 4.89µm and the
GSD is 2.5 cm.
a) 17.9 m
b) 179 m
c) 1,789 m
d) 17,895 m
17. Using a DSLR with a focal length of 35mm on a sUAS,
calculate the flying height if the pixel size is 4.89µm and the
GSD is 2.5 cm.
a) 17.9 m
b) 179 m
c) 1,789 m
d) 17,895 m

https://cdn-images-1.medium.com/max/1600/1*gTaMQi8tipdRYF_vuoe4fg.png
ab f
S= =
AB H'
4.89μm 35mm
= =
2.5cm Xm

X=
(35mm )(2.5cm )
4.89μ.

=
(35mm )(2.5cm ) https://americansecuritytoday.com/wp-
content/uploads/2016/04/UAS.jpg

0.00489mm
= 179m
A. The advantages of a fixed-wing UAS are ______.
a) a, b, d
a. Can operate indoors and in
b) b, d, f tighter environments
c) c, d, f b. Higher maneuverability
d) c, e, f c. Higher payload capacity
d. Longer flight lines
e. Only a small take-off and landing
is required
f. Stability better in poor weather
conditions.
Unmanned Vehicle Systems for Geomatics by C. Armenakis and P. Patias

A. The advantages of a fixed-wing UAS are ______.


a) a, b, d
a. Can operate indoors and in
b) b, d, f tighter environments
c) c, d, f b. Higher maneuverability
d) c, e, f c. Higher payload capacity
d. Longer flight lines
e. Only a small take-off and landing
is required
The incorrect answers are
advantages of rotary UASs – f. Stability better in poor weather
Another rotary advantage is conditions.
the ability to hover.
B. In the UAV, the relative orientation of the axes of the IMU
and the sensor’s axis are called the ____________.
a) boresight angles
b) Euler angles (roll, pitch and yaw)
c) lever arm parameters
d) orientation angles (κ, ϕ and ω)
Unmanned Vehicle Systems for Geomatics by C. Armenakis and P. Patias

B. In the UAV, the relative orientation of the axes of the IMU


and the sensor’s axis are called the ____________.
a) boresight angles
b) Euler angles (roll, pitch and yaw)
c) lever arm parameters
d) orientation angles (κ, ϕ and ω)
C. Larger amounts of overlap and sidelap are generally used in
UASs than typical photogrammetric projects to __________.
a) compensate for camera swaying during the flight
b) compensate for platform instability
c) enhance the accuracy by decreasing the air base
d) maintain consistent interior orientation
Unmanned Vehicle Systems for Geomatics by C. Armenakis and P. Patias

C. Larger amounts of overlap and sidelap are generally used in


UASs than typical photogrammetric projects to __________.
a) compensate for camera swaying during the flight
b) compensate for platform instability
c) enhance the accuracy by decreasing the air base
d) maintain consistent interior orientation

Larger amounts of overlap and sidelap


also allow for multi-viewing geometry.
18. Sensor accuracy for sUASs is generally __________ than for
those sensors mounted in conventional aircraft.
a) higher
b) lower
c) indistinguishable
d) more stable
S. Rhee & T. Kim, “Automated DSM Extraction from UAV Images and Performance Analysis, ISPRS
Canada, 2015

18. Sensor accuracy for sUASs is generally __________ than for


those sensors mounted in conventional aircraft.
a) higher
b) lower
c) indistinguishable
d) more stable
S. Rhee & T. Kim, “Automated DSM Extraction from UAV Images and Performance Analysis, ISPRS
Canada, 2015

Sensor Accuracy
• For UAVs, sensor accuracy was lower than those mounted on the
aerial or satellite images
• The stability of the platform also low
• Initial EOPs caused serious match errors and poor results
19. Exterior orientation parameters for precise geometric
analysis from UAS imagery are
a) automatically provided in the direct georeferencing.
b) difficult to obtain.
c) easily extracted using conventional AT adjustment
packages.
d) stable when combined with on-board GPS-IMU.
H. Eisenbeiss, “UAV Photogrammetry”, Institut fur Geodasie und Photogrammetrie, 2009

19. Exterior orientation parameters for precise geometric


analysis from UAS imagery are
a) automatically provided in the direct georeferencing.
b) difficult to obtain.
c) easily extracted using conventional AT adjustment
packages.
d) stable when combined with on-board GPS-IMU.
20. Once a sUAS operator completes the Aeronautical
Knowledge Test they still have to finish the FAA Airman
Certificate and/or Rating Application to get their remote
pilot certificate.
a) True
b) False
An Early Survey of Best Practices for the Use of Small Unmanned Aerial Systems by the Electric Utility Industry by R. Lusk & W. Monday
http://info.ornl.gov/sites/publications/Files/Pub73072.pdf

20. Once a sUAS operator completes the Aeronautical


Knowledge Test they still have to finish the FAA Airman
Certificate and/or Rating Application to get their remote
pilot certificate.
a) True
b) False

https://wingsmqy.wordpress.com/2009/12/19/plastic-not-paper/
UAS Testing
https://www.uasvision.com/2016/05/27/p

• Test results should be available within 48hr ge-testing-uas-to-inspect-electric-and-gas-


infrastructure/

• Application should be validated within 10 days


• Printing instructions for a temporary airman certificate sent to
applicant
• Temporary airman certificate good for 120 days
• PAA provides permanent Remote Pilot Certificate within 120 days
• Regulation is not applicable to hobbyists
21. Binoculars can be used by sUAS operators in order for the
sUAS to be within vision.
a) True
b) False
An Early Survey of Best Practices for the Use of Small Unmanned Aerial Systems by the Electric Utility Industry by R. Lusk & W. Monday
http://info.ornl.gov/sites/publications/Files/Pub73072.pdf

21. Binoculars can be used by sUAS operators in order for the


sUAS to be within vision.
a) True
b) False

According to FAA, the sUAS must be close enough to the PIC and
person manipulating the flight control so that it can be seen using
normal vision w/o any other device other than corrective lenses

PIC is Pilot In Command – for sUAS refers to


Remote Pilot In Command
22. A sUAS cannot fly higher than ___ feet above a structure’s
uppermost limit.
An Early Survey of Best Practices for the Use of Small Unmanned Aerial Systems by the Electric Utility Industry by R. Lusk & W. Monday
http://info.ornl.gov/sites/publications/Files/Pub73072.pdf

22. A sUAS cannot fly higher than 400 feet above a structure’s
uppermost limit.

Maximum altitude of a sUAS is 400’ AGL or, if higher


than 400’ AGL, remain within a 400’ radius of a
structure; fly no higher than 400’ above the
structure’s uppermost limit.

http://www.adroitlawyers.com.au/can-i-be-fined-for-flying-my-drone/
23. To obtain a remote pilot certificate the individual must be
vetted by the ___________________________________.
An Early Survey of Best Practices for the Use of Small Unmanned Aerial Systems by the Electric Utility Industry by R. Lusk & W. Monday
http://info.ornl.gov/sites/publications/Files/Pub73072.pdf

23. To obtain a remote pilot certificate the individual must be


vetted by the Transportation Security Administration (TSA).

https://www.pmenv.com/articles/do-drones-
have-place-phase-i-esa-process
24. A UAS pilot who is certified in Canada can operate a sUAS in
U.S. NAS.
a) True
b) False
An Early Survey of Best Practices for the Use of Small Unmanned Aerial Systems by the Electric Utility Industry by R. Lusk & W. Monday
http://info.ornl.gov/sites/publications/Files/Pub73072.pdf

24. A UAS pilot who is certified in Canada can operate a sUAS in


U.S. NAS.
a) True
b) False

Until international standards are developed, foreign-certified


UAS pilots will be required to obtain a FAA-issued remote pilot
certificate with a sUAS rating.
25. Even though FAA airworthiness certification is not required
the remote PIC must complete a ________________.
An Early Survey of Best Practices for the Use of Small Unmanned Aerial Systems by the Electric Utility Industry by R. Lusk & W. Monday
http://info.ornl.gov/sites/publications/Files/Pub73072.pdf

25. Even though FAA airworthiness certification is not required


the remote PIC must complete a
pre-flight check .

Pre-flight check is required to ensure that the


sUAS is in a condition for safe operation

Includes checking the communications link


between the UAS and the control station
26. A COA is a ___________________________.
26. A COA is a Certificate of Authorization.

http://slideplayer.com/slide/10863194/
References
• Unmanned Vehicle Systems for Geomatics by C. Armenakis and P.
Patias, Whittles Publishing
• Introduction to Unmanned Aircraft Systems 2nd edition by D.
Marshall, R. Barnhart, E. Shappee, M. Most (Eds.), CRC Press,
2016
• Introduction of UAV Systems (Aerospace Series), 4th edition by P.
Fahlstrom and T. Gleason, Wilely, 2012
• Fundamentals of Remote Sensing and Airphoto Interpretation,
5th edition by T. Avery and G. Berlin, Prentice Hall, 1992
• Manual of Photogrammetry, 6th edition by J. McGlone (Editor),
ASPRS, 2015
• Remote Sensing and Image Interpretation, 7th edition by T.
Lillesand R. Kiefer and J. Chipman, Wiley, 2015
• Elements of Photogrammetry: With Applications in GIS, 4th
edition by P. Wolf, B. DeWitt and B. Wilkerson, McGraw Hill, 2014.
• Manual of Airborne Topographic Lidar by M. Renslow (Ed.),
ASPRS, 2015
For Information Contact
Robert Burtch, PS, CP (Ret)

robert.burtch66@gmail.com
rburtch@charter.net
REMOTE SENSING
Preparation for ASPRS Certification: General Knowledge Workshop
DEFINITION OF REMOTE SENSING
Science and art of obtaining information about an object,
area, or phenomenon through the analysis of data acquired by
a device that is not in contact with the object, area, or
phenomenon under investigation

Remote Sensing and Image Interpretation, 3rd edition


T. Lillesand and R. Kiefer
1. The visible portion of the electromagnetic
spectrum is defined between __________.
a) 0.05 – 0.4 µm
b) 0.4 – 0.7 µm
c) 0.7 – 1.0 µm
d) 10 – 70 µm
e) 10 – 1000 mm
1. The visible portion of the electromagnetic
spectrum is defined between __________.
a) 0.05 – 0.4 µm
b) 0.4 – 0.7 µm
c) 0.7 – 1.0 µm
d) 10 – 70 µm
e) 10 – 1000 mm
2. An active sensing system is one which
__________.
a) can be activated/deactivated on demand
b) can only be operated manually
c) continuously collects image data
d) emits it own source of energy
2. An active sensing system is one which
__________.
a) can be activated/deactivated on demand
b) can only be operated manually
c) continuously collects image data
d) emits it own source of energy
SENSOR
•The right answer is emits its own source of energy
• An example often used is a camera with a flash attachment
•A passive sensor is one that relies on naturally available
energy
• Using the camera analogy, a passive sensor is just a camera – relies
on the sun’s energy
3. When sunlight interacts with the
atmosphere the short (blue)
wavelengths are redirected more
than the other visible wavelengths.
This is called __________.
a) absorption
b) Mie scatter
c) non-selective scatter
d) Rayleigh scatter
3. When sunlight interacts with the
atmosphere the short (blue)
wavelengths are redirected more
than the other visible wavelengths.
This is called __________.
a) absorption
b) Mie scatter
c) non-selective scatter
d) Rayleigh scatter
Scattering
• Redirection of electromagnetic energy by
particles
• Amount depends on
• sizes of particles
• abundance
• wavelength of radiation
• depth of atmosphere were energy is
traveling
RAYLEIGH SCATTER
 Can see blue sky
 At sunrise/sunset, sun’s rays travel through longer
atmospheric path therefore we see only the less scattered,
longer wavelength of orange and red
 Primary cause of “haze” in imagery
 In color, gives a bluish-gray cast to image
PHYSICS OF REMOTE SENSING
• Absorption
• Results in loss of energy
• Involved absorption of energy at a given
wavelength
PHYSICS OF REMOTE SENSING
• Non-selective scatter
• Caused when diameter of particles are much larger than the energy of
wavelength being used
• Scatter non-selective with respect to wavelength
• In visible wavelengths, equal quantities of blue, green, red light are scattered,
making fog and clouds appear white
PHYSICS OF REMOTE SENSING
• Mie scatter
• When diameter of atmospheric particles about the same size as wavelength
• Water vapor and dust are primary causes
• Significant in slightly overcast atmospheric conditions
4. A film characteristic curve
describes
a) film opacity to film
transmittance
b) image density to film
exposure level
c) spectral reflectance
d) total radiant exitance
from the surface of a
material
4. A film characteristic curve
describes
a) film opacity to film
transmittance
b) image density to film
exposure level
c) spectral reflectance
d) total radiant exitance
from the surface of a
material
FILM
b is the correct answer • a deals with opacity and
• Also called the D-log E curve transmittance
describing density to the relative • Opacity – one measure of
log exposure darkness or lightness at a given
• Differs for film types, point
manufacturing batches, and for • Transmittance – ratio of light
films of same batch passing through the film to the
• Affected by handling, total light incident upon the film
manufacturing, storage, and
processing conditions
FILM
• c – spectral reflectance • d – refers to the Stefan Boltzman
• Ratio of energy of wavelength λ Law
incident upon the object • Amount of energy an object
radiates is a function of the
surface temperature of the object
5. Surface reflectors that are flat surfaces giving
mirror-like reflections are known as __________
a) diffuse reflectors
b) energy quantum reflectors
c) Lambertian reflectors
d) specular reflectors
5. Surface reflectors that are flat surfaces giving
mirror-like reflections are known as __________
a) diffuse reflectors
b) energy quantum reflectors
c) Lambertian reflectors
d) specular reflectors

http://micro.magnet.fsu.edu/primer/java/scienceo
pticsu/reflection/specular/index.html
PHYSICS OF REMOTE SENSING
• Specular reflector is the correct • Answers diffuse and Lambertian
answer reflectors are the same answers
• With this kind of surface, the • These are rough surfaces which
angle of reflection is reflects the energy uniformly in
approximately equal to the angle all directions
of incidence
6. The ratio of emitted radiation to absorbed
radiation flux is the same for all blackbodies at
the same temperature is known as __________.
a) Kirchhoff’s law
b) Planck’s law
c) Stefan-Boltzmann law
d) Wien’s displacement law
6. The ratio of emitted radiation to absorbed
radiation flux is the same for all blackbodies at
the same temperature is known as __________.
a) Kirchhoff’s law
b) Planck’s law
c) Stefan-Boltzmann law
d) Wien’s displacement law
• Emissivity of true
Kirchhoff’s Law blackbody is 1 while
perfect reflector is 0
(whitebody)
• Ratio of emitted radiation
to absorbed radiation flux • In nature, all objects fall
is same for all blackbodies between -- graybodies
at same temperature
• Useful measure of
• ε = M/Mb effectiveness of object as
radiator of EM energy
• Objects with high
• where ε is emissivity and emissivities absorb high
M is emittance of an proportions of incident
object and Mb is emittance radiation and then redirect
of blackbody this energy
Planck’s Law
• Electromagnetic
energy absorbed and
emitted in discrete Q=hv
units called quanta or
photons • Q = radiant energy
• Size proportional to • h = Planck’s constant
frequency • v = frequency
• Explains photoelectric https://ozonedepletiontheory.info/ImagePages/Plancks-law-
frequency1.html

effect - impact of
quanta on some metal
surfaces causes the
emission of electrons
https://www.britannica.com/bio
graphy/Max-Planck/images-
videos/media/462888/18070
Stefan-Boltzmann Law
http://www.pas.roch
ester.edu/~blackman
/ast104/radiation.ht
ml

• Hot blackbodies emit more energy


per unit area than do cool
blackbodies

W = σT 4

• W = total emitted radiation


• σ = Stefan-Boltzmann constant
= 5.6697 x 10-8 Watts m-2K-4
• T = Temperature (in Kelvin) Boltzmann
• Radiation emitted from blackbody
proportional to 4th power of its
absolute temperature
Wien’s Displacement Law
• As objects become hotter,
the wavelength of
maximum emittance shifts C
to shorter wavelengths λ=
T
• As blackbody becomes
hotter, wavelength of max
emittance shifts to shorter λ = wavelength of maximum
wavelength spectral radiant exitance, µm
C = 2,897.8 µm°K
T = temperature, °K
7. Differentiation of conifers and deciduous tree
stands is best done by using a sensor that is
sensitive to what part of the electromagnetic
spectrum?
a) Blue
b) Green
c) Red
d) Near-infrared
7. Differentiation of conifers and deciduous tree
stands is best done by using a sensor that is
sensitive to what part of the electromagnetic
spectrum?
a) Blue
b) Green
c) Red
d) Near-infrared
8. A sun-synchronous orbit is one
which
a) crosses the equator at the same
time for each pass.
b) crosses the equator at a time
difference based on the satellite
period.
c) crosses the equator at each
pass earlier than the previous
pass based on the period.
d) remains in the sky fixed to the
earth.
8. A sun-synchronous orbit is one
which
a) crosses the equator at the same
time for each pass.
b) crosses the equator at a time
difference based on the satellite
period.
c) crosses the equator at each
pass earlier than the previous
pass based on the period.
d) remains in the sky fixed to the
earth.
PHYSICS OF REMOTE SENSING

• Crossing the equator at the same time for each pass is the correct
answer
• Advantage is continuity in solar illumination
• Constant relation between orbital node and direction of sun is maintained
PHYSICS OF REMOTE SENSING
• d) remains fixed with the earth rotation is called a geostationary or
geosynchronous orbit
• Useful for communications and weather satellites
• Fixed with the earth
• Satellite placed in equatorial plane of the earth at a speed equivalent to the
earth’s rotation
9. A good signal to noise ratio is one
where
a) the noise levels are
approximately equal to
the brightness of the
target.
b) the noise levels are
small relative to the
brightness of the target.
c) the noise levels are
large relative to the
brightness of the target.
d) the noise levels are
eliminated thereby
enhancing target
brightness.
9. A good signal to noise ratio is one
where
a) the noise levels are
approximately equal to
the brightness of the
target.
b) the noise levels are
small relative to the
brightness of the target.
c) the noise levels are
large relative to the
brightness of the target.
d) the noise levels are
eliminated thereby
enhancing target
brightness.
10. The ability of a sensor to define fine wavelength
intervals is called __________.
a) spatial resolution
b) radiometric resolution
c) spectral resolution
d) temporal resolution
10. The ability of a sensor to define fine wavelength
intervals is called __________.
a) spatial resolution
b) radiometric resolution
c) spectral resolution
d) temporal resolution
RESOLUTION
Spectral Resolution
• Number and size of spectral regions or bands covered
• High Spectral Resolution – ability to sense even relatively small bands
within a given resolution
• Examples
• Landsat TM – 7 spectral bands in range 0.4 – 12.5 μm
• SPOT High Resolution Visible (HRV) – 0.5 – 0.89 μm
• NASA’s Airborne Visible & Infrared Imaging Spectrometer (AVIRIS) – 224 contiguous bands
between 0.4 – 2.5 μm (0.01μm or 10nm bandwidths)
• Multispectral – sensor records energy in few bands
• Hyperspectral – sensor records energy in hundreds of bands (10
nanometers or smaller)
RESOLUTION
Spatial Resolution
• Ability to record spatial data
• Minimum distance between objects that allow them to be differentiated
• Deals with the fineness of the spatial detail visible in an image
• Function of size of detector, focal length, sensor altitude, system configuration
• Film – determined by photosensitive silver grains and usually defined by lines per
millimeter
• Digital sensor – ground area falling within IFOV – Instantaneous Field of View of
single detector or pixel size
Resolution
Radiometric Resolution
• Ability of imaging sensor to record levels of brightness
• Example: number of bits recorded
• Ability to discriminate very slight energy differences
• Examples:
• Landsat MSS – 6 bits originally
• Landsat TM – 8 bits
• IKONOS – 11 bits
RESOLUTION

Temporal Resolution
• Time interval sensor repeats data collection at same location
• Important when studying phenomenon over extended or specific
amount of time
• Land-use change, seasonal crop growth, recent disasters
RESOLUTION
• Information content of sensor data directly proportional to spatial,
spectral, temporal and radiometric resolution
• Careful selection of the resolutions increases discrimination between
features to obtain ground features, etc.
11. Tone is one of the basic characteristics utilized in
air photo interpretation. Tone refers to the
a) configuration or outline of an individual
object.
b) repetition of the general form or
relationship.
c) relative brightness or color of objects.
d) occurrence of certain features relative to
others.
11. Tone is one of the basic characteristics utilized in
air photo interpretation. Tone refers to the
a) configuration or outline of an individual
object.
b) repetition of the general form or
relationship.
c) relative brightness or color of objects.
d) occurrence of certain features relative to
others.
INTERPRETATION
Configuration or outline refers to shape - outline of an
object
Repetition of general form refers to pattern –
repetition of form
Frequency of certain relation is called association –
relation of features relative to others
Elements of Visual Interpretation

From: http://emyshoran.blogspot.com/2010/12/elements-of-visual-interpretation.html
Photo Interpretation
Tone refers to the relative brightness or color of objects in an image. Generally, tone is
the fundamental element for distinguishing between different targets or features.
Variations in tone also allows the elements of shape, texture, and pattern of objects to
be distinguished.

Shape refers to the general form, structure, or outline of individual objects. Shape can
be a very distinctive clue for interpretation. Straight edge shapes typically represent
urban or agricultural (field) targets, while natural features, such as forest edges, are
generally more irregular in shape, except where man has created a road or clear cuts.
Farm or crop land irrigated by rotating sprinkler systems would appear as circular
shapes.
Photo Interpretation
Size of objects in an image is a function of scale. It is important to assess the size of
a target relative to other objects in a scene, as well as the absolute size, to aid in
the interpretation of that target. A quick approximation of target size can direct
interpretation to an appropriate result more quickly. For example, if an interpreter
had to distinguish zones of land use, and had identified an area with a number of
buildings in it, large buildings such as factories or warehouses would suggest
commercial property, whereas small buildings would indicate residential use.
Pattern refers to the spatial arrangement of visibly discernible objects. Typically an
orderly repetition of similar tones and textures will produce a distinctive and
ultimately recognizable pattern. Orchards with evenly spaced trees, and urban
streets with regularly spaced houses are good examples of pattern.
Photo Interpretation
Texture refers to the arrangement and frequency of tonal variation in particular areas of an image. Rough
textures would consist of a mottled tone where the grey levels change abruptly in a small area, whereas
smooth textures would have very little tonal variation. Smooth textures are most often the result of
uniform, even surfaces, such as fields, asphalt, or grasslands. A target with a rough surface and irregular
structure, such as a forest canopy, results in a rough textured appearance. Texture is one of the most
important elements for distinguishing features in radar imagery.
Shadow is also helpful in interpretation as it may provide an idea of the profile and relative height of a
target or targets which may make identification easier. However, shadows can also reduce or eliminate
interpretation in their area of influence, since targets within shadows are much less (or not at all)
discernible from their surroundings. Shadow is also useful for enhancing or identifying topography and
landforms, particularly in radar imagery.
Association takes into account the relationship between other recognizable objects or features in
proximity to the target of interest. The identification of features that one would expect to associate with
other features may provide information to facilitate identification. In the example given above,
commercial properties may be associated with proximity to major transportation routes, whereas
residential areas would be associated with schools, playgrounds, and sports fields. In our example, a lake
is associated with boats, a marina, and adjacent recreational land.
12. The thematic mapper sensor on board Landsat
records _____ spectral bands.
a) 4
b) 5
c) 6
d) 7
12. The thematic mapper sensor on board Landsat
records _____ spectral bands.
a) 4
b) 5
c) 6 The Enhanced Thematic
d) 7 Mapper thermal has 2
gains thus yielding 8
spectral bands.
13. The scan line corrector
compensates for the
a) along track motion of the
spacecraft during active
cross track scans.
b) skew error in the image.
c) small overlap between
scenes north and south.
d) temporal registration
problem in scenes
acquired at the same
location.
13. The scan line corrector
compensates for the
a) along track motion of the
spacecraft during active
cross track scans.
b) skew error in the image.
c) small overlap between
scenes north and south.
d) temporal registration
problem in scenes
acquired at the same
location.
14. A false color Landsat MSS
composite image consists
of
a) blue, green, and red
spectral bands.
b) green, red, and
infrared spectral
bands.
c) green, red, and
thermal spectral
bands.
d) green, infrared, and
thermal spectral
bands.
14. A false color Landsat MSS
composite image consists
of
a) blue, green, and red
spectral bands.
b) green, red, and
infrared spectral
bands.
c) green, red, and
thermal spectral
bands.
d) green, infrared, and
thermal spectral
bands.
15. What kind of product processing of remotely
sensed imagery is characterized by putting the
data into map coordinates?
a) Geometric correction
b) Basic georeferencing
c) Full georeferencing
d) Orthorectified
e) Enhanced
15. What kind of product processing of remotely
sensed imagery is characterized by putting the
data into map coordinates?
a) Geometric correction
b) Basic georeferencing
c) Full georeferencing
d) Orthorectified
e) Enhanced
PROCESSING
• Geometric correction is raw data resampled to correct for geometric
errors due to earth’s rotation and incidence angle of the sensor
• Basic georeferencing, or geocoding,– data put into map coordinates
and then customer’s choice of map projection
• Fully georeferencing/geocoding – data corrected using control points
• Orthorectified – horizontal and vertical distortion removed from
image
• Enhanced – computerized algorithms used to increase quality of
imagery – most commonly, contrast stretch
16. The process of resampling to correct for image
geometry based on a weighted average of the
four nearest input pixels is called __________.
a) bilinear interpolation
b) cubic convolution
c) linear stretch
d) nearest neighbor resampling
16. The process of resampling to correct for image
geometry based on a weighted average of the
four nearest input pixels is called __________.
a) bilinear interpolation
b) cubic convolution
c) linear stretch
d) nearest neighbor resampling
Processing
https://image.slidesharecdn.com/remotesensing-111209014340-phpapp02/95/remote-
sensing-44-728.jpg?cb=1323396456

• Bilinear interpolation
• Average is more natural look
• Brightness values of original
image lost
• Range of brightness values in
output different than from input
• Decreases spatial resolution
Processing https://image.slidesharecdn.com/remotesensing-111209014340-phpapp02/95/remote-sensing-45-
728.jpg?cb=1323396456

• Cubic Convolution
• Weighted average of
nearest 16 neighbors
https://image.slidesharecdn.com/remotesensing-111209014340-
phpapp02/95/remote-sensing-43-728.jpg?cb=1323396456

Processing
• Nearest neighbor
• Use just nearest neighbor
• Simple and preserves original scene
values
• Creates positional errors
• Linear stretch
• Contrast enhancement
http://www.nrcan.gc.ca/sites/w
• Converts DN values into new distribution ww.nrcan.gc.ca/files/earthscien
ces/images/resource/tutor/fun
using new min and max values dam/images/linstre.gif
17. One remote sensing data format can be
described as: band1, line 1; band 2, line 1;
band 3, line 1; band 4, line 1; band 1, line 2,
etc. This format is referred to as _________.
a) BLS
b) BIL
c) BIP
d) BSQ
17. One remote sensing data format can be
described as: band1, line 1; band 2, line 1;
band 3, line 1; band 4, line 1; band 1, line 2,
etc. This format is referred to as _________.
a) BLS
b) BIL
c) BIP
d) BSQ

http://ecomputernotes.com/images/Single-bit-error.jpg
DATA FORMAT
• Band interleaved by pixel • Band sequential
• Line 1, pixel 1, band 1 • Band 1
• Line 1, pixel 1, band 2 • Band 2
• Line 1, pixel 1, band 3 • Band 3
• Line 1, pixel 1, band 4 • Band 4
• Line 1, pixel 2, band 1
• etc.

http://www.r-s-c-c.org/node/229
18. The phenomenon whereby a terrain feature
appears to have a steeper slope than they have
in nature on the near-range and shallower
slopes than in nature on the far range is called
_________.
a) radar shadow
b) radar polarization
c) radar layover
d) radar foreshortening
e) radar penetration
18. The phenomenon whereby a terrain feature
appears to have a steeper slope than they have
in nature on the near-range and shallower
slopes than in nature on the far range is called
_________.
a) radar shadow
b) radar polarization
c) radar layover
d) radar foreshortening
e) radar penetration
RADAR
Foreshortening
• Ground Dist. AB’ = B’C
• Radar positions by time delay so
it perceives positions in slant
range
• Slant range positions differ ab ≠
bc
FORESHORTENING
• Results from fact that relief displacement is towards
the radar
• Different than photogrammetry
• Range increases more slowly than ground coordinates
on slopes facing towards radar making them tend to
be bunched relative to planimetric view
• Opposite occurs on slopes facing away – tend to
expand out when compared to plan view
RADAR • Radar shadow
• Occurs when radar
cannot see a part of
terrain
• Terrain occulted by
other parts of terrain
or objects in scene
• In image, shadow area
will appear dark and
any signal in these cells
are due to thermal
noise
RADAR
• b) polarization
• Orientation of electromagnetic field
• Either horizontally or vertically polarized
• HH image or like-polarized – transmits horizontally
polarized signal and receives horizontally polarized
return signal
• HV or cross-polarized mode
• Images that depolarize will be bright on HV image and
dark or dark gray on corresponding HH image
• Causes: rough surface wrt signal wavelength and
volume scattering from inhomogeneous medium like
signal penetrating below the soil surface
Layover
RADAR • Near Range - top of tall
object closer to antenna
• Echo from top reaches
antenna before echo from
base
• Occupy different image
positions in slant-range
• Points arranged with
increasing ground
coordinates appear
reversed in radar image
LAYOVER
• Geometrically – happens when slope of terrain is greater than the
incident angle makes with respect to the vertical
• More important:
• For interferometric radar systems – layover causes a loss of useful signal and
then precludes the determination of elevation in layover regions
RADAR
• e) penetration
• Assessed by specifying skin depth –
depth at which signal strength is
reduced to 1/e of surface magnitude
• Skin depth increases with increase
wavelength and in absence of
moisture
• Greater at steeper angles and
decreases as incidence angle
increases https://upload.wikimedia.org/wikipedia/commons/9/9c/LINE21.jpg
• Max penetration at near-range and
minimum at far-range
19. A coordinate transformation from the image
space to the frequency domain is called a
_________.
a) Fourier Transform
b) high pass filter
c) Laplace operator
d) Sobel operator
19. A coordinate transformation from the image
space to the frequency domain is called a
_________.
a) Fourier Transform
b) high pass filter
c) Laplace operator
d) Sobel operator
http://www.r-s-c-c.org/node/227
High Pass Filter
• The high pass filter allows high frequency data to pass
through, suppressing low frequency data. High pass
filtering can be useful for finding edges, enhancing lines
and edges, or sharpening an image. Small box cars will
enhance small features and details. Large box cars will
allow large features to pass through, suppressing or
eliminating smaller features.

https://isis.astrogeology.usgs.go
v/IsisWorkshop/index.php/The_
Power_of_Spatial_Filters
Laplace Operator
• Laplacian Operator is a derivative operator which is
used to find edges in an image. It is a second order
derivative mask.

Original Image Laplace Operator


Sobel Operator
• The Sobel Edge filter is use to detect edges based
applying a horizontal and vertical filter in sequence.
Both filters are applied to the image and summed to
form the final result

http://www.geo-
informatie.nl/courses/grs20306/
course/Schedule/Digital-filters-
RS-new.pdf
20. A method of supervised classification where
clusters are defined by a range of values
(minimum and maximum) within
multidimensional data space is called a
_________.
a) Baye’s classifier
b) maximum likelihood classifier
c) minimum distance classifier
d) parallelepiped classifier
20. A method of supervised classification where
clusters are defined by a range of values
(minimum and maximum) within
multidimensional data space is called a
_________.
a) Baye’s classifier
b) maximum likelihood classifier
c) minimum distance classifier
d) parallelepiped classifier
PARALLELEPIPED CLASSIFICATION
• Box decision rule or level-slice procedures
• Based on ranges of values within training data to define boundaries
• Standard deviations can also be used to define boundaries
• Unclassified pixels assigned class if they fall within regions defined by
training data assigned to the appropriate category
PARALLELEPIPED CLASSIFICATION

• Advantages
• Accurate
• Direct
• Simple
• Example of corn
& hay
• Gives elongated,
slanted clouds of
observations
PARALLELEPIPED CLASSIFICATION
• Disadvantages
• Spectral regions for informational categories may intersect
• Overlap caused by correlation or high covariance
• Poorly described by rectangular decision rule
• Training data may underestimate actual ranges and leave large areas
unassigned to informational categories
• Areas classified as “not sure” or “unknown”
• Regions not uniformly occupied by pixels in each category – problem with
mixed pixels

• While often used, may not be most effective choice of image


classification
PARALLELEPIPED CLASSIFICATION

• Can modify
classifier by
changing the
rectangular
category
ranges into
steps
MINIMUM DISTANCE TO MEAN CLASSIFIER
• Uses central values of spectral data in assigning pixels to
informational class
• Each cluster represented by a centroid, often defined as mean
• Compute distance from unknown pixel to each category mean
• Unknown pixel assigned to “closest” class
• If distance greater than some max value, classified as unknown
MINIMUM
DISTANCE TO
MEAN
CLASSIFIER

• Insensitive to degrees of variance in data and simple to


implement
• Not always accurate; no means of accounting for variability of
classes; some classes may overlap at edges
MAXIMUM LIKELIHOOD CLASSIFIER
• Realize that extreme values for spectral classes overlap in
informational categories
• Data contains mixed pixels
• Strategy is to choose class that maximizes the probability of a correct
classification
• Uses training data to estimate mean and variances of classes to which
it assigns probabilities
• Variance accounts for variability within a category
MAXIMUM LIKELIHOOD CLASSIFIER
• Compute probability density functions – 1 function
for each category
MAXIMUM LIKELIHOOD CLASSIFIER
• Create
equiprobability
contours
• Shape of contour
expresses
sensitivity of
classifier to
covariance
21. A dot grid is placed over a conifer stand
consisting of 480 ha. The analyst counts 87 dots
within the area. Another area of conifers is
shown to be 65 dots. The acreage of this second
conifer stand is __________.
a) 359 ha
b) 460 ha
c) 502 ha
d) 642 ha
21. A dot grid is placed over a conifer stand
consisting of 480 ha. The analyst counts 87 dots
within the area. Another area of conifers is
shown to be 65 dots. The acreage of this second
conifer stand is __________.
a) 359 ha 480 ha x
b) 460 ha =
87 dots 65 dots
c) 502 ha
 480 ha 
d) 642 ha x =  65 dots = 359 ha
 87 dots 
Dot Grid

http://ibis.geog.ubc.ca/courses/geob373/labs/scale_and_area_measurement_files/scale_and_area_dotgrid_method_mapuseandanalysis.jpg
22. Water __________ energy in the near infrared
region.
a) absorbs
b) reflects
c) scatters
d) transmits
22. Water __________ energy in the near infrared
region.
a) absorbs
b) reflects
c) scatters
d) transmits

http://baby.indstate.edu/gerstt/rscc/f7chutfnw.jpg
23. Plotting reflectance values on a simple diagram
constructed using the brightness of two spectral
bands as orthogonal axes is called a(n)
a) Euclidean distance diagram.
b) principal component plot.
c) scatter diagram.
d) X, Y-plot.
23. Plotting reflectance values on a simple diagram
constructed using the brightness of two spectral
bands as orthogonal axes is called a(n)
a) Euclidean distance diagram.
b) principal component plot.
c) scatter diagram.
d) X, Y-plot.
Accuracy Assessment
Image Classification

Conifer Deciduous Agriculture Water Σ

Conifer 317 61 2 35 415


Reference Data

Deciduous 23 120 4 29 176

Agriculture 0 0 60 0 60

Water 0 0 0 8 8

Σ 340 180 66 72 659


24. The producer’s accuracy for deciduous is
______.
a) 18%
b) 32%
c) 66%
d) 68%
24. The producer’s accuracy for deciduous is
______.
a) 18%
b) 32%
c) 66%
d) 68%
Producer’s Accuracy
• Number correctly classified divided by total number of pixels in a
class
• For deciduous
• 23 deciduous classified as conifer
• 120 deciduous classified as deciduous
• 4 deciduous classified as agriculture
• 29 deciduous classified as water
• 176 is total number of deciduous pixels in reference data
Producer’s Accuracy
• Producer’s Accuracy (Pc):
No. correctly classsified 120
pa Deciduous = = = 0.682 or 68%
Correct no. of reference pixels 176

• Errors of omission – assignment of pixels of Category A to another


category
Error of Omission = 1 − Producer' s Accuracy
= 1 − pa Deciduous
= 1 − 0.68 = 0.32 or 32%
25. The pixels correctly classified (overall accuracy)
is _______.
a) 23%
b) 51%
c) 77%
d) 86%
25. The pixels correctly classified (overall accuracy)
is _______.
a) 23%
b) 51%
c) 77%
d) 86%
Accuracy Assessment
• Pixels correctly classified also called overall accuracy
• Sum diagonal elements divided by total number of pixels

 1 k
317 + 120 + 60 + 8
p = ∑ cii =
n i =1 659
= 0.766 or 77%
RECOMMENDED REFERENCES
• Manual of Remote Sensing, 2nd edition, ASPRS (out of
print)
• Replaced by a series of volumes comprising the 3rd edition.
6 volumes thus far published
• 4th edition is an e-manual
• Remote Sensing and Image Interpretation, 7th
edition, by Thomas Lillesand, Ralph Kiefer, Jonathan
Chipman
• Introduction to Remote Sensing, 5th edition, by
James Campbell and Randolph Wynne
• Remote Sensing of the Environment: An Earth
Resource Perspective, 2nd edition by John Jensen
From: http://lidar-america.com/wp-content/uploads/2014/03/LiDAR-Escaneo-
Ejemplo.jpg

Lidar
Preparation for ASPRS Certification: General Knowledge Workshop
Detroit Free Press, Sunday,
February 4, 2018
1. The shorter the wavelength the _____________.
a) higher the energy
b) lower the energy
c) less likely the energy gets absorbed
d) stability of the signal become unstable
L. Graham. “The Advent of 3D Color?”, Lidar Magazine, July/Aug 2016.

1. The shorter the wavelength the _____________.


a) higher the energy
b) lower the energy
c) less likely the energy gets absorbed
d) stability of the signal become unstable
L. Graham. “The Advent of 3D Color?”, Lidar Magazine, July/Aug 2016.

Laser wavelength
• Short wavelengths
• More effective in detecting small particles like cloud
droplets & drizzle drops
• Also partially absorbed by these same particles (called
attenuation)
• EMR energy related to wavelength in inverse way
• Imagine packing more wavelength cycles into the same space if the
distances between crests become shorter.
• Important property of lasers are that the emitted light is a
very narrow wavelength
• Since a specific color implies a specific wavelength, this means the
laser is one pure color.
2. Increasing the range of a lidar system ________
the point cloud density.
Lee Hathcock, 2016, “Evaluation of Current and Upcoming LIDAR Systems”,
http://www.gri.msstate.edu/publications/docs/2016/09/15022LIDAR_V5.pdf

2. Increasing the range of a lidar system decreases


the point cloud density.

http://felix.rohrba.ch/en/2015/point-density-and-point-spacing/
3. Detectable surface remnants of buildings, trees,
towers, telephone poles or other elevated features in
a bare earth model are referred to as _____________.
a) artifacts
b) back scatter
c) echo
d) point dropout
Renslow, M., editor, 2012. Manual of Airborne Topographic Lidar

3. Detectable surface remnants of buildings, trees,


towers, telephone poles or other elevated features in
a bare earth model are referred to as _____________.
a) artifacts
b) back scatter
c) echo
d) point dropout
Renslow, M., editor, 2012. Manual of Airborne Topographic Lidar

• Back scatter – electromagnetic energy that is reflected back towards


its source by terrain or particles in the atmosphere
• Echo - each of the multiple returns from an emitted laser pulse in a
multiple-pulse-return laser scanning system (e.g., first, intermediate
… last)
• Point dropout – laser pulse for which no energy was returned to the
sensor
4. When the light beam diameter increases with distance
from the aperture, this is referred to as __________.
a) beam divergence
b) range
c) swath width
d) time interval meter
Renslow, M., editor, 2012. Manual of Airborne Topographic Lidar

4. When the light beam diameter increases with distance


from the aperture, this is referred to as __________.
a) beam divergence
b) range
c) swath width
d) time interval meter
Renslow, M., editor, 2012. Manual of Airborne Topographic Lidar

• Range – distance between the laser aperture and the detected object
or surface
• Swath width – the width of the survey area covered by a complete
sweep of the scanner which is related to flying height and scanner
angle
• Time interval meter – the method used to time-stamp the pulse
transmission and reception points to determine the time difference
between the pulse transmission and reception, and resulting in the
range distance and position of the reflected surface
5. The maximum operational range and measurement accuracy
of a laser rangefinder is available with an instrument with a
_______ laser pulse repetition frequency and ________ laser
pulse.
a) higher; narrower
b) higher; wider
c) smaller; narrower
d) smaller; wider
Renslow, M., editor, 2012. Manual of Airborne Topographic Lidar

5. The maximum operational range and measurement accuracy


of a laser rangefinder is available with an instrument with a
_______ laser pulse repetition frequency and ________ laser
pulse.
a) higher; narrower
b) higher; wider
c) smaller; narrower
d) smaller; wider
6. Which of the following is not a laser scanning system
classification?
a) Continuous laser system
b) Non-contact optical profilometer
c) Pulsed laser system
d) Time interval meter
Renslow, M., editor, 2012. Manual of Airborne Topographic Lidar

6. Which of the following is not a laser scanning system


classification?
a) Continuous laser system
b) Non-contact optical profilometer
c) Pulsed laser system
d) Time interval meter
Renslow, M., editor, 2012. Manual of Airborne Topographic Lidar

• Non-contact optical profilometer – in topographic applications, based


on a combination of laser light stripe generator and a video camera.
The laser source is separate from the video camera. Laser light is
visible and follows along the target surface continuously creating a
surface profile
• Time interval meter - the method used to time-stamp the pulse
transmission and reception points to determine the time difference
between the pulse transmission and reception, and resulting in the
range distance and position of the reflected surface
7. The horizontal error in lidar derived elevation data is
largely a function of ________.
a) a, b, d
b) a, c, d a)Attitude (angular orientation
c) b, c, d error from INS
d) c, d, e b)Echo from return signal
c)Flying height
d)Positional error derived from
GNSS
e)Pulse repetition error
http://www.asprs.org/PAD-Division/ASPRS-POSITIONAL-ACCURACY-STANDARDS-FOR-DIGITAL-
GEOSPATIAL-DATA

7. The horizontal error in lidar derived elevation data is


largely a function of ________.
a) a, b, d
b) a, c, d a)Attitude (angular orientation
c) b, c, d error from INS
d) c, d, e b)Echo from return signal
c)Flying height
d)Positional error derived from
GNSS
e)Pulse repetition error
8. A laser scanning system that produces a zig-zag line on
the surface of the target area is a(n) __________
system.
a) fiber scanning
b) line scanning
c) oscillating mirror
d) Palmer scanning
Renslow, M., editor, 2012. Manual of Airborne Topographic Lidar

8. A laser scanning system that produces a zig-zag line on


the surface of the target area is a(n) __________
system.
a) fiber scanning
b) line scanning
c) oscillating mirror
d) Palmer scanning
Renslow, M., editor, 2012. Manual of Airborne Topographic Lidar

• Fiber scanning –laser pulse transmitted into one of the fibers


arranged in a circle
• Line scanning – mirror is rotated at a constant velocity in one
direction by a motor resulting in measured points being parallel lines
on the ground
• Palmer scanning – produces an elliptical pattern with redundant data
that can be used for calibration
https://uta-ir.tdl.org/uta-ir/bitstream/handle/

Scanning Systems
10106/24975/Hu_uta_2502M_12971.pdf?sequence=1

a) Oscillating mirror
b) Line scan
c) Palmer scan
d) Fiber scan
9. The accuracy of laser rangefinder technology is very
high because of ___________ of the laser light beam.
a) high divergence
b) low divergence
c) no divergence
d) stable divergence
Renslow, M., editor, 2012. Manual of Airborne Topographic Lidar

9. The accuracy of laser rangefinder technology is very


high because of ___________ of the laser light beam.
a) high divergence
b) low divergence
c) no divergence
d) stable divergence
10. A lidar unit capable of detecting a single photon is
called a ___________.
a) flash lidar
b) full-waveform lidar
c) Geiger mode lidar
d) pulsed lidar
Renslow, M., editor, 2012. Manual of Airborne Topographic Lidar
https://pbs.twimg.com/media/CIHdiKUUcAEG7DW.jpg

10. A lidar unit capable of detecting a single photon is


called a ___________. http://www.fosternav.net/project.htm

a) flash lidar
b) full-waveform lidar
c) Geiger mode lidar
d) pulsed lidar
Renslow, M., editor, 2012. Manual of Airborne Topographic Lidar

lidar
• Flash lidar (Flash imaging radar) – entire scene within the
sensor’s field of view (FOV) is imaged with a single flash of
the laser on the order of a nanosecond producing a 3D image
• Full-waveform lidar –digitize the entire back scattered laser
pulse at a very high sampling rate yielding a complete record
of each received signal
• Geiger mode lidar – operate by sending out multiple, lower
energy, smaller width pulses at a high PRF rate, and
accumulating the number of detections at each APD element
over a specified number of pulses
• Pulsed lidar – uses pulsed laser at a particular wavelength
and pulse energy. A short pulse originates at the laser & is
returned from the target. Time of flight measured. Called
discrete return system
APD – Avalanche Photodiode
11. The __________________ is the base specification
for the National interagency 3D Elevation Program
a) Global Multi-Resolution Terrain
Elevation Model
b) Lidar Base Specification
c) National Elevation Dataset
d) Shuttle Radar Topography Mission
Heidemann, Hans, Lidar Base Specification, Version 1.3, Feb 2018
https://pubs.usgs.gov/tm/11b4/pdf/tm11-B4.pdf

11. The __________________ is the base specification


for the National interagency 3D Elevation Program
a) Global Multi-Resolution Terrain
Elevation Model
b) Lidar Base Specification
c) National Elevation Dataset
d) Shuttle Radar Topography Mission
• Global Multi-Resolution Terrain Elevation Data
• Collaboration between USGS & NGA to develop an enhanced global elevation model
for global & continental scale applications
• Provides global coverage from 84°N to 90°S
• 3 resolutions: 30” (arc) about 1 km spacing; 15” (arc) about 500m spacing; 15”
(arc) about 250m spacing
• https://catalog.data.gov/dataset/global-multi-resolution-terrain-elevation-data-national-geospatial-data-asset-ngda

• National Elevation Dataset


• 1” (arc) [~30m] resolution
• Derived from diverse source data processed to common coordinate system and
unit of vertical measure
• https://www.sciencebase.gov/catalog/item/4fcf8fd4e4b0c7fe80e81504

• Shuttle Radar Topography Mission


• Elevation data derived from C-band radar data
• Uses interferometric C-band Spaceborne Imaging Radar to generate elevation data
• https://catalog.data.gov/dataset/shuttle-radar-topography-mission-1-arc-and-3-arc-second-digital-terrain-elevation-data-void-fi
12. According to the Lidar Base Specification, the
minimum acceptable quality level for 3DEP collection
is _______
a) QL0
b) QL1
c) QL2
d) QL3
Heidemann, Hans, Lidar Base Specification, Version 1.3, Feb 2018
https://pubs.usgs.gov/tm/11b4/pdf/tm11-B4.pdf

12. According to the Lidar Base Specification, the


minimum acceptable quality level for 3DEP collection

http://gisuser.com/2016/03/3d-elevation-program-3dep-broad-
is _______
a) QL0
b) QL1

agency-announcement-baa-2016-awards/
c) QL2
d) QL3
Heidemann, Hans, Lidar Base Specification, Version 1.3, Feb 2018
https://pubs.usgs.gov/tm/11b4/pdf/tm11-B4.pdf

Lidar Quality Level Specifications

Aggregate nominal pulse spacing & density Absolute vertical accuracy for lidar & DEMs
Relative vertical accuracy for lidar swath
Minimum digital elevation model cell size
data
13. The uncertainty in coordinates of mapped points with
respect to the geodetic datum at the 95% confidence
level is _______.
a) local accuracy
b) network accuracy
c) positional accuracy
d) relative accuracy
Heidemann, Hans, Lidar Base Specification, Version 1.3, Feb 2018
https://pubs.usgs.gov/tm/11b4/pdf/tm11-B4.pdf

13. The uncertainty in coordinates of mapped points with


respect to the geodetic datum at the 95% confidence
level is _______.

https://en.wikipedia.org/wiki/Lidar#/media/File:Effigy_mounds_lidar.jpg
a) local accuracy
b) network accuracy
c) positional accuracy
d) relative accuracy
Heidemann, Hans, Lidar Base Specification, Version 1.3, Feb 2018
https://pubs.usgs.gov/tm/11b4/pdf/tm11-B4.pdf

Accuracy
• Local accuracy
• Uncertainty in point coordinates with respect to coordinates of other directly
coordinated, adjacent points at the 95% confidence level.
• Positional accuracy
• Accuracy at the 85% confidence level of the position of features with respect
to the horizontal and vertical datum
• Relative accuracy
• Measure of variation in point-to-point accuracy in a data set
• In lidar, may mean positional agreement between points within a swath,
adjacent swaths within a lift, adjacent lifts within a project, or between
adjacent projects
14. Surface nonreflectance of a lidar signal from
reflection or scatter results in a(n) _______.
a) aggregated combined signal
b) combined mixed signal
c) data void
d) multipath signal
Heidemann, Hans, Lidar Base Specification, Version 1.3, Feb 2018
https://pubs.usgs.gov/tm/11b4/pdf/tm11-B4.pdf

14. Surface nonreflectance of a lidar signal from


reflection or scatter results in a(n) _______.
a) aggregated combined signal Data voids shown in red.
b) combined mixed signal https://www.researchgate.net/figure/LiDAR-points-bordering-
on-voids-red-Figure-3-displays-an-example-of-
proximity_fig1_288184244

c) data void
d) multipath signal
15. Helicopters are generally not employed for projects
with the following conditions:
a) Higher point densities with overlapping
flight lines
b) High vertical accuracy
c) Need for maneuverability
d) Requirements for high point densities
Renslow, M., editor, 2012. Manual of Airborne Topographic Lidar

15. Helicopters are generally not employed for projects


with the following conditions:
a) Higher point densities with overlapping
flight lines
b) High vertical accuracy
c) Need for maneuverability
d) Requirements for high point densities
Renslow, M., editor, 2012. Manual of Airborne Topographic Lidar

Lidar project planning


• Helicopter: • Fixed wing:
• Need for maneuverability such as • Ability to acquire wide areas of
along coverage at higher flight altitudes
• Requirement for high point & speed (cost effectiveness)
density (obtained from slower • Option for high point densities
aircraft speed) with overlapping flight lines
• High vertical accuracies (obtained • Vertical accuracy sufficient for
from lower flight heights) most large to medium scale
mapping applications
16. For best or maximum water penetration with a lidar
system a laser wavelength in the range of
___________ nanometers should be used.
a) 10 – 400
b) 450 – 495
c) 495 – 570

http://aucillaresearchinstitute.org/?page_id=134
d) 620 - 750
16. For best or maximum water penetration with a lidar
system a laser wavelength in the range of
___________ nanometers should be used.
a) 10 – 400
b) 450 – 495
c) 495 – 570
d) 620 - 750
17. In general, which project scenario requires significant
overlap to ensure adequate ground sampling?
a) Corridor projects
b) Forested projects
c) Mountainous projects
d) Urban projects
Renslow, M., editor, 2012. Manual of Airborne Topographic Lidar

17. In general, which project scenario requires significant


overlap to ensure adequate ground sampling?
a) Corridor projects http://gis.stackexchange.com/questions/94288/how-to-automatically-

b) Forested projects
check-for-lidar-flight-line-overlaps

c) Mountainous projects
d) Urban projects
Renslow, M., editor, 2012. Manual of Airborne Topographic Lidar

Lidar projects
• Corridor projects – present flight plan options such as flying
with a helicopter, breaking flight lines, and/or flying multiple
parallel lines to cover the alignment
• Forested projects – require consideration of lidar instrument
settings to maximize canopy penetration and/or maximize
identification of tree crown or understory
• Mountainous projects – require large variations in flight
height which in turn affect the post spacing, swath width,
laser footprint size, and accuracy
• Urban projects – require flight planning with significant
overlap to ensure adequate sampling of the ground between
tall buildings
18. The footprint of the laser upon the ground is affected
by
a) a, b, c
b) a, c, e a)Aircraft speed
c) b, c, e b)Beam divergence
d) b, d, e c)Flight height
d)Point density
e)Scan angle
Renslow, M., editor, 2012. Manual of Airborne Topographic Lidar

18. The footprint of the laser upon the ground is affected


by
a) a, b, c
b) a, c, e a)Aircraft speed
c) b, c, e b)Beam divergence
d) b, d, e c)Flight height
d)Point density
e)Scan angle
19. This lidar error is represented by an upward or
downward arc appearance when profiling
perpendicular to the flight direction within scans
a) Pitch slope error
b) Roll
c) Torsion/scan factor error
d) Yaw
e) Z-bias
Renslow, M., editor, 2012. Manual of Airborne Topographic Lidar

19. This lidar error is represented by an upward or


downward arc appearance when profiling
perpendicular to the flight direction within scans
a) Pitch slope error
b) Roll http://forsys.cfr.washington.edu/JFSP
06/lidar_technology.htm

c) Torsion/scan factor error


d) Yaw
e) Z-bias
Renslow, M., editor, 2012. Manual of Airborne Topographic Lidar

Lidar error
• Roll – the rising or dipping of the aircraft’s wing
• Pitch – the movement of the aircraft’s nose either up or down
• Yaw – aircraft moving towards the left or right while in flight
• Pitch slope error – when the pitch is correct at nadir of a scan
and offset at the edge of a scan
• Torsion or scan factor error – represented by an upward or
downward arc appearance when profiling perpendicular to
flight direction within scans (swath data); the correction for this
error will be consistent for all scans of a given mission
• Z-bias – elevation offset between the lidar data set and the
control used to check the lidar data
20. Because it is easier to obtain tighter variables as they
are more easily detected in flat areas, __________
accuracy should be used.
a) absolute
b) consolidated
c) fundamental
d) relative
Renslow, M., editor, 2012. Manual of Airborne Topographic Lidar

20. Because it is easier to obtain tighter variables as they


are more easily detected in flat areas, __________
accuracy should be used.
a) absolute
b) consolidated
c) fundamental
d) relative
FEMA Procedure Memorandum No. 61 – Standards for Lidar and Other
High Quality Digital Topography

Accuracy
• Consolidated vertical accuracy (CVA) • Fundamental vertical accuracy (FVA) –
– result of a test of the accuracy of value by which vertical accuracy can
vertical check-points (Z-values) be equitably assessed and compared
consolidated for 2 or more of the among datasets. Determined with
major land cover categories, vertical check points located only in
representing both open terrain and open terrain where very high
other land cover categories probability that sensor will detect
ground surface
Note that ASPRS Positional Accuracy Standards for Digital Geospatial Data have
replaced CVA with Vegetated Vertical Accuracy (VVA) which is used by the NDEP
(National Digital Elevation Program) guidelines for vertical accuracy at the 95th
percentile in all land cover categories combined. FVA has been replaced by Non-
vegetated Vertical Accuracy (NVA) which is used by NDEP guidelines for vertical
accuracy at the 95th confidence level in open terrain only where errors should
approximate a normal distribution.
21. The effect of a laser scan profile upon the
reconstructed surface over sloped terrain due
to a GPS timing bias is
a) an elevation error in the reconstructed
surface.
b) a positional shift in the reconstructed
surface.
c) a rotational shift about the flight axis.
d) both a positional shift and elevation error
in the reconstructed surface.
21. The effect of a laser scan profile upon the
reconstructed surface over sloped terrain due
to a GPS timing bias is
a) an elevation error in the reconstructed
surface.
b) a positional shift in the reconstructed
surface.
c) a rotational shift about the flight axis.
d) both a positional shift and elevation error
in the reconstructed surface.
LASER ALTIMETRY (LIDAR)
• Positional shift causes a corresponding elevation error
• Elevation error shown as:
∆Z = s ' tan γ

s'

Z
22. Bathymetric lidar systems use the ___________
portion of the electromagnetic spectrum.
a) ultraviolet-blue
b) blue-green
c) green-red
d) red-infrared
22. Bathymetric lidar systems use the ___________
portion of the electromagnetic spectrum.
a) ultraviolet-blue
b) blue-green
c) green-red
d) red-infrared
23. If a lidar unit pointing is in the nadir direction and it is located
1,000m AGL, having a sensor orientation of 5 mrad and a yaw
angle of 15o, the linear error on the ground is about ______.
a) 5 m
b) 259 m
c) 284 m
d) 1035 m
23. If a lidar unit pointing is in the nadir direction and it is located
1,000m AGL, having a sensor orientation of 5 mrad and a yaw
angle of 15o, the linear error on the ground is about ______.
a) 5 m
b) 259 m
c) 284 m
e = H' sin δ
d) 1035 m
= (1,000 m) sin 0.005rad

= 5m

Note that 1 mrad = 0.001 rad


24. In the flight planning stage, lidar point density is usually
determined based on ______.
a) a, b, e
b) a, c, f
c) b, c, d
a)application
d) c, e, f b)distance from base station
c)features in area
d)lidar error
e)type of terrain
f) vegetation
Renslow, M., editor, 2012. Manual of Airborne Topographic Lidar

24. In the flight planning stage, lidar point density is usually


determined based on ______.
a) a, b, e
b) a, c, f
c) b, c, d
a)application
d) c, e, f b)distance from base station
c)features in area
d)lidar error
e)type of terrain
f) vegetation
25. Since most vegetation typically occurs near
__________ this complicates the classification
process.
a) drainage/water bodies
b) edges of farm fields
c) housing/subdivisions
d) roads/highways
Renslow, M., editor, 2012. Manual of Airborne Topographic Lidar

25. Since most vegetation typically occurs near


__________ this complicates the classification
process.
a) drainage/water bodies
b) edges of farm fields
c) housing/subdivisions
d) roads/highways
RECOMMENDED REFERENCES
• Manual of Airborne Topographic Lidar, by
M. Renslow (editor)
• Topographic Laser Ranging and Scanning,
by J. Shan and C. Toth (editors)
• Airborne and Terrestrial Laser Scanning, by
G. Vosselman and H-G, Maas (editors)
• Making Spatial Decisions Using GIS and
Lidar: A Workbook, by Kathryn Keranen and
Robert Kolvoord (Example of a number of
lidar/GIS workbooks)
Geographic Information
Systems/Land information
Systems
Preparation for ASPRS Certification: General Knowledge Workshop
1. A well defined GIS should be able to
a) select an area by either area or theme
b) link or merge one data set with another
c) search an area for a particular characteristic
d) all of the above
1. A well defined GIS should be able to
a) select an area by either area or theme
b) link or merge one data set with another
c) search an area for a particular characteristic
d) all of the above
GIS DEFINITION
• In addition, the GIS • Different definition
should also • Depends on who gives it,
• Analyze data’s spatial background, point of view
characteristics • Likely to change as technology and
• Quickly and economically applications develop
update data
• Generally 3 components
• Model data and assess
alternatives • Computer system
• Utilize spatially referenced data
• Manage and analyze data
GIS Definition
• Computer-based system to integrate, manipulate, extract, synthesize,
and display geospatial data in either raster or attributed vector format
• Principal goal is to synthesize and/or extract geospatial information in
response to a geospatial query
2. The first comprehensive digital street map of
the entire US is
a) BLM - Public Land Survey Plats.
b) Bureau of Census - TIGER system.
c) DoD - Digital Chart of the World.
d) FWS - digital National Wetlands
Inventory.
e) USGS - 1:24,000 Quad Maps.
2. The first comprehensive digital street map of
the entire US is
a) BLM - Public Land Survey Plats.
b) Bureau of Census - TIGER system.
c) DoD - Digital Chart of the World.
d) FWS - digital National Wetlands
Inventory.
e) USGS - 1:24,000 Quad Maps.
DEMOGRAPHIC DATA
• Bureau of Census’ TIGER system
• Topologically Integrated Geographic Encoding and Referencing system
• Derived from scanned USGS 1:100,000-scale maps
• Bureau of Census assigned geographic attributes
• USGS file shortcomings
• No feature names
• Outdated maps
TIGER’S BASIC STRUCTURE
• 0-cell – point of intersection,
termination, or change in category
of one or more 1-cell 0-cell
ell
• 1-cell – line or arc defined by 2 0-
c
1-

1-c
0-cell

ell
cells 1-
cell
1-
ce
ll 0-cell

• 2-cell – smallest polygon enclosed 0-cell 2-cell

by a chain of 1-cells 1-c


ell

0-cell
TIGER SHAPE POINTS
• Shape points provide coordinate values that describe the shape of
features that are not straight

End Point
Shape Point
Shape Point

Shape Point
Lake Road

Shape Point

Shape Point

End Point
PLSS
• Public Land Survey System
• Map of the surveys performed for
describing land for transfer out of
Federal ownership
• Covers most of US but not the original
13 colonies, Texas and Hawaii

• Divided land into townships


• 6 miles square
• Townships divided into sections
• 1 mile square
USGS 1:24,000 QUAD MAPS
• Paper maps covering 7.5’ x 7.5’ of
area
• Largest scale general purpose
series map by USGS
• Topographic base map series
• Shows a wealth of detail:
boundaries, airports, contours,
hydrography, railroads, PLSS
boundaries, vegetation, water, etc.
FWS’s DIGITAL NATIONAL WETLANDS
INVENTORY
• U.S. Fish and Wildlife Service
• Digitized from 1:24,000-scale wetlands maps into
topologically correct data files containing ground
planimetric coordinates and wetland attributes
• Need to check on availability in area
• Maps compiled from National Aerial Photography
Program 1:40,000-scale and National High Altitude
Photography program 1:58,000 or 1:80,000-scale
aerial photography
• Mapped all photo-interpretable wetlands
DoD’s DIGITAL CHART OF THE WORLD
• Global coverage of topographic
information equivalent in detail to
1:1,000,000-scale map
• Shows major road and rail networks,
major drainage systems, major utility
networks, major airports, contours,
coastlines, international boundaries,
and populated places
• Topologically structure, thematically
layered feature data in Vector Product
Format
3. The property whereby an object does not
change under transformation such as
stretching or bending is ___________.
a) affine transformation
b) conformal transformation
c) Helmert transformation
d) topology
3. The property whereby an object does not
change under transformation such as
stretching or bending is ___________.
a) affine transformation
b) conformal transformation
c) Helmert transformation
d) topology
SPATIAL PROPERTIES
• Topology • Affine, conformal and Helmert
• Further characteristics of topology transformations involve
are independence of any projecting data in one
coordinate system, independence coordinate system into another
of scale of measurement
4. The measurement scale that identifies
meaningful differences between numbers,
but without a real origin, is called a(n)
___________.
a) fractal scale
b) interval scale
c) nominal scale
d) ordinal scale
4. The measurement scale that identifies
meaningful differences between numbers,
but without a real origin, is called a(n)
___________.
a) fractal scale
b) interval scale
c) nominal scale
d) ordinal scale
MEASUREMENT SCALE
• Nominal scale
• Used to establish identity
• Does feature exist or not exist
• Ex: house, road, river
• Ordinal scale
• Establishes rank between features
without values
• Ex: large, medium, small city; hot
and cold
MEASUREMENT SCALE
• Interval scale
• Rank ordering where differences
between ranks is meaningful
• Ex: 10o F isotherms
• Ratio scale – sometimes linked
with interval scale
• Same as the interval scale except
there is an absolute zero
• Ex: Kelvin temperature, elevations
• Fractal scale
• Relates to the “ruggedness” of a
feature such as a contour line
5. Given the following raster data set, the
Manhattan distance from A to B is
___________.
a) 8. 3 units
b) 4.2 units
c) 6 units
d) 9 units
5. Given the following raster data set, the
Manhattan distance from A to B is
___________.
a) 8. 3 units
b) 4.2 units
c) 6 units
d) 9 units
DISTANCE MEASUREMENT
There are 4 basic types of raster measurement
• Euclidean distance – Pythagorean theorem

AB = AC 2 + CB 2 = 4.2 units
• Manhattan distance – how one would navigate blocks in a street
• 3 blocks over and 3 blocks up = 6 blocks
• Taxi driver distance – similar to Manhattan distance but need to
consider impedances like 1-way streets
DISTANCE MEASUREMENT
• Proximity distance – set up concentric zones
about a point/cell
• Sometimes called spread
6. Given the following Venn diagram, what is the
Boolean operator?
a) A and B
b) A not B
c) A or B
d) A xor B
6. Given the following Venn diagram, what is the
Boolean operator?
a) A and B
b) A not B
c) A or B
d) A xor B
BOOLEAN OPERATOR
A or B looks like
A and B looks like

A xor B looks like


EXAMPLE BOOLEAN OPERATOR
• 2 Maps with 4 categories each
Land Use Slope
B 10
A
20
30
D C
40

• Find the area that has land use A and slope 30


EXAMPLE BOOLEAN OPERATOR
• Simple overlay of two raster files

10
A B • Approach
20 10 New • Reclassify Land Use Map
Polygons • 1 = A; 0 = B, C, D
30 Created • Reclassify Slope Map
D C
• 1 = 30
40
0 = 10, 20, 40
- Result: 2 binary images
EXAMPLE BOOLEAN OPERATOR
• Use Boolean AND operation –
multiply
• 0x0=0
• 0x1=0
Binary Image
1 Meeting Criteria • 1x0=0
of Land Use A • 1x1=1
0 and Slope 30
• Binary image with only 1’s and
0’s
• Reclassify
7. That part of a quality report specified by the
Metadata Standard which includes a description
of the source material from which the data were
derived and the methods of derivation is
referred to as __________.
a) attribute accuracy
b) completeness
c) lineage
d) logical consistency
e) positional accuracy
7. That part of a quality report specified by the
Metadata Standard which includes a description
of the source material from which the data were
derived and the methods of derivation is
referred to as __________.
a) attribute accuracy
b) completeness
c) lineage
d) logical consistency
e) positional accuracy
SDTS
• Lineage describe
• Basic data
• Where it came from
• What manipulations were involved in its creation
• What is the geographic reference being used
• etc.
SDTS SPATIAL DATA QUALITY
• Positional accuracy • Attribute accuracy
• Describes compliance to a • Used for measures on a
standard continuous scale
• Preferred method of testing is by • Can use deductive estimates (i.e..
comparison to an independent Good, poor, etc) and
survey of higher accuracy (see misclassification matrix
ASPRS Accuracy Standards for
Large Scale Maps)
SDTS SPATIAL DATA QUALITY
• Logical consistency • Completeness
• How good are the relationships • Are all necessary graphic and non-
encoded in the data? graphic data in data base?
• Do lines intersect where intended? • Includes information about
• Are any lines entered twice? selection criteria, definitions used
• Area all areas completely and other relevant mapping rules
described? • Describes exhaustiveness of a set
• Are there any overshoots or of features: spatial and taxonomic
undershoots? (attribute) properties
• Are any polygons too small or lines
too close
SPATIAL DATA TRANSFER STANDARD (SDTS) –
See Metadata Standard
• Defines:
• Terminology
• Spatial data transfer specification
• Methods for reporting digital cartographic data quality
• Topographic and hydrographic entity terms and definitions
• Allows users to transfer digital spatial data sets between dissimilar computing systems
• Approved as Federal Information Processing Standard (FIPS) Publication 173, known
as FIPSPUB 173-1, 1994.
• FIPS version has been superseded by version, known as ANSI NCITS 320-1998 and
was ratified by the American National Standards Institute (ANSI) June 9, 1998.
• Withdrawn in 2014 – GML replaces SDTS and satisfies encoding requirements
Geography Markup Language
• GML 3.2.1 approved & published as ISO standard
• ISO – International Organization for Standardization
• Mandated standard in DoD Information Technology Standards Repository
(DISR)
• Originally developed by Open Geospatial Consortium (OGC)
• International consortium of companies, government agencies, universities
• Open consensus process to develop publically available interface standards
Geography Markup Language
• Open, vendor-neutral eXtensible
Markup Language (XML)
• Transport & storage of geographic
information
• Supports spatial and non-spatial
properties of objects
• Extensible – extendable
• Supports definition of profiles (proper
subsets) of the full GML capabilities
• Core concept is feature
• Feature is abstraction of phenomenon in
real world
• Every feature has a feature type
8. Simplification of spatial data on a map in order
to maintain clarity is called __________.
a) feature representation
b) generalization
c) map annotation
d) spatial referencing system
8. Simplification of spatial data on a map in order
to maintain clarity is called __________.
a) feature representation
b) generalization
c) map annotation
d) spatial referencing system
SPATIAL ANALYSIS
• Feature representation
• Selection of features
• Generalization • Points – represent features too
• All data generalized by some small to be area
extent • What is a point and an area depends
• Grain size of film, resolution of on scale
remote sensing sensor, depiction • Lines – linear features with no
of features on map, existing data width
like Census data • What is a line and area depends on
• Function of purpose and scale scale
• Area – features with length and
width
• Often called polygon
SPATIAL ANALYSIS
• Spatial referencing system
• Locates features on earth’s surface
• Ex: geographic coordinates, rectangular coordinates
• Map annotation
• Using map keys, legends, text to facilitate use of map
9. An algorithm defined by extending a line from a
central point and counting the number of times
it intersects a polygon boundary is used for
__________.
a) buffer generation
b) point-in-polygon analysis
c) polygon-on-polygon analysis
d) proximity analysis
9. An algorithm defined by extending a line from a
central point and counting the number of times
it intersects a polygon boundary is used for
__________.
a) buffer generation
b) point-in-polygon analysis
c) polygon-on-polygon analysis
d) proximity analysis
SPATIAL ANALYSIS
• Point-in-polygon analysis
• If no. of crossings odd then point is in polygon,
otherwise outside
SPATIAL ANALYSIS
• Polygon-on-polygon analysis
• Overlay two polygons
• example
SPATIAL ANALYSIS
• Union • Identity • Intersect
10. A database management system in which data
are stored in a matrix structure of rows and
columns is called a __________.
a) flat file model
b) hierarchical model
c) network model
d) relational model
10. A database management system in which data
are stored in a matrix structure of rows and
columns is called a __________.
a) flat file model
b) hierarchical model
c) network model
d) relational model
DATABASE MANAGEMENT
• Relational model
• One of the more common db models
• Each row is a record and each column a field
• A record consists of the attribute data about a feature
• A field displays a particular attribute that exists for each feature
• Gives the user a lot of flexibility in the kinds of queries they may wish
to perform
DATABASE MANAGEMENT
• Hierarchical model
• Based on a parent-child or one-to-
many relationship
• Efficient model if
• Relationships can be clearly defined
• Queries are standard, can be
predefined, and follow those
relationships
DATABASE MANAGEMENT
• Network model
• Groups records of the same type
into conceptual files
• One-to-many or many-to-many
relationships
• Allows more than one record in a
file to be associated with more
than one record in another file
DATABASE MANAGEMENT
• Sequential and flat file models
are the same
• Similar in structure as relational k
except that search is performed
using a “key” field
k

• Search is performed by the key


k
k = key field
field then the attribute data are k

extracted k

k
Database Management http://www.colorado.edu/geography/gcraft/notes/datacon/datacon.bak13

Structure Advantages Disadvantages


Flat Files • Fast data retrieval • Difficult to process multiple values of a data item
• Simple structure and easy to program • Adding new data categories requires reprogramming
• Slow data retrieval without the key

Hierarchical • Adding and deleting records is easy • Pointer path restricts access
Files • Fast data retrieval through higher level • Each association requires repetitive data in other
records records
• Multiple associations with like records in • Pointers require large amount of computer storage
different files

Relational Files • Easy access and minimal technical training • New relations can require considerable processing
for users • Sequential access is slow
• Flexibility for unforeseen inquiries • Method of storage on disks impacts processing time
• Easy modification and addition of new • Easy to make logical mistakes due to flexibility of
relationships, data, and records relationships between records
• Physical storage of data can change without
affecting relationships between records
Database
Management

http://www.colorado.edu/geography/gcraft/notes/datacon/gif/flatfile.gif

http://www.colorado.edu/geography/gcraft/notes/datacon/gif/hierarch.gif

http://www.colorado.edu/geography/gcraft/notes/datacon/gif/relafile.gif
11. Cache memory is
a) a temporary storage memory designed to
lower execution time and increase
performance.
b) an auxiliary storage such as floppy disks, CD-
ROMs, tape drives, etc.
c) the high speed data paths between
peripheral computer equipment.
d) the main memory unit providing high speed
access to the data.
11. Cache memory is
a) a temporary storage memory designed to
lower execution time and increase
performance.
b) an auxiliary storage such as floppy disks, CD-
ROMs, tape drives, etc.
c) the high speed data paths between
peripheral computer equipment.
d) the main memory unit providing high speed
access to the data.
COMPUTER HARDWARE
• When the CPU executes a • The main memory in the
program, it first looks for the computer is called random
data or instructions in the cache access memory or core memory
memory
• If not found, it goes to the
auxiliary devices • High speed data paths are called
buses
12. The binary number 10101101 is equivalent to
the base 10 number _______.
a) 5
b) 10
c) 173
d) 241
12. The binary number 10101101 is equivalent to
the base 10 number _______.
a) 5
b) 10
c) 173
d) 241
COMPUTER HARDWARE

• The solution is as follows

7 6 5 4 3 2 1 0
2 2 2 2 2 2 2 2
1 0 1 0 1 1 0 1
128 0 32 0 8 4 0 1 =173
Computer Hardware
Inversely,

27 26 25 24 23 22 21 20

128 64 32 16 8 4 2 1

1 0 1 0 1 1 0 1

• One 128; the remainder 173 – 128 = 45; Thus no 64;


Then one 32 with a remainder 45 – 32 = 13; Thus no 16;
Then one 8 with a remainder of 13 – 8 = 5; Then one 4
with a remainder of 5 – 4 = 1; There are no 2s but one 1.
Computer
Take theHardware
same number 173 to base 4 (Quad)

44 43 42 41 40

256 64 16 4 1

0 2 2 3 1

• No 256; There are 2 64s in 173 and the remainder 173 – (64
x 2) = 45; There are 2 16s in 45 so the remainder is 45 – (16
x 2) = 13; There are 3 4s in 13 with a remainder 13 – (4 x 3)
= 1; Finally there is 1 one. The answer is 17310 = 22314.
13. A __________ is a digitized aerial photograph
that has been differentially rectified to remove
image displacement due to camera tilt and
terrain relief.
a) DLG
b) DOQ
c) DXF
d) VPF
13. A __________ is a digitized aerial photograph
that has been differentially rectified to remove
image displacement due to camera tilt and
terrain relief.
a) DLG
b) DOQ
c) DXF
d) VPF
File Formats
• DLG – Digital line graph
• digital representation of point, line and area features from USGS quadrangles
• DOQ – Digital orthophoto quad
• DXF – Drawing interchange file
• Neutral format data exchange by AutoCAD
• VPF – Vector product format
• Used by DoD for digital vector data
14. An adjacency matrix indicates the
a) accuracy level in classification.
b) coordinate transformation results.
c) relationships between nodes and chains.
d) smoothness of a cartographic line.
14. An adjacency matrix indicates the
a) accuracy level in classification.
b) coordinate transformation results.
c) relationships between nodes and chains.
d) smoothness of a cartographic line.

http://www.colorado.edu/geography/g
craft/notes/datacon/datacon.bak13
DATA MODEL
Relationship between nodes and chains
• 1 indicates from_node (row) to_node (column) for chain
15. A TIN model is a
a) raster image where pixel attributes are
defined by gradient.
b) topographic map created by direct
measurement of contours.
c) topological map structure used to measure
flow.
d) triangulation mesh comprised of discrete
sample points over an area.
15. A TIN model is a
a) raster image where pixel attributes are
defined by gradient.
b) topographic map created by direct
measurement of contours.
c) topological map structure used to measure
flow.
d) triangulation mesh comprised of discrete
sample points over an area.
16. Metadata provides the user with
knowledge of data such as
a. does the data meet
__________. specific needs
a) a, b, c, d b. how can data be
b) a, b, c, e accessed
c) a, c, d, e c. how is information
organized in the
d) b, c, d, e computer
d. what data are available
e. where can the data be
found
16. Metadata provides the user with
knowledge of data such as
a. does the data meet
__________. specific needs
a) a, b, c, d b. how can data be
b) a, b, c, e accessed
c) a, c, d, e c. how is information
organized in the
d) b, c, d, e computer
d. what data are available
e. where can the data be
found
17. __________ involves manually digitizing features
and points on a computer monitor, often using
digital imagery as the backdrop.
a) Heads up digitizing
b) Overlay digitization
c) Scanning
d) Tablet digitizing
17. __________ involves manually digitizing features
and points on a computer monitor, often using
digital imagery as the backdrop.
a) Heads up digitizing
b) Overlay digitization
c) Scanning
d) Tablet digitizing
18. An example of a continuous data structure is
__________.
a) business store locations
b) elevations
c) incidents of robberies
d) population density
18. An example of a continuous data structure is
__________.
a) business store locations
b) elevations
c) incidents of robberies
d) population density
DATA STRUCTURE
• Continuous data
• Can occur at every possible location in an area like elevations, temperature,
etc.
• Discrete data
• Data occurring at only selected locations
19. The process of predicting a value at a location
using known values occurring within a site is
_________.
a) extrapolation
b) interpolation
c) tessellation
d) transformation
19. The process of predicting a value at a location
using known values occurring within a site is
_________.
a) extrapolation
b) interpolation
c) tessellation
d) transformation
SPATIAL ANALYSIS
• Extrapolation – predicting a value to a location outside
of the area of interest
• Tessellation – subdivision of an area into polygonal
tiles that cover the area
• Transformation – process of converting data from an
original form to another form
20. Orientation of the slope of a surface in a
certain direction is referred to as __________.
a) aspect
b) cross-section
c) profile
d) steepness
20. Orientation of the slope of a surface in a
certain direction is referred to as __________.
a) aspect
b) cross-section
c) profile
d) steepness
SPATIAL ANALYSIS
• Steepness refers to rise over run
• Profile is the trace of a vertical line through a surface
• Cross-section is process of sampling points
perpendicular to a line through a site
• Statistical surface – data used to represent a surface
or operate on a surface
21. A region mapped at 1:25,000 will only occupy
______ as much as the map space when mapped
at 1:100,000.
a) 1/2th
b) 1/4th
c) 1/8th
d) 1/16th
21. A region mapped at 1:25,000 will only occupy
______ as much as the map space when mapped
at 1:100,000.
a) 1/2th
b) 1/4th
c) 1/8th
d) 1/16th
22. One method of random-to-grid interpolation is
the ___________ wherein the interpolated
heights are established by fitting a surface
through the complete data set.
a) global method
b) local method
c) patchwise method
d) pointwise method
22. One method of random-to-grid interpolation is
the ___________ wherein the interpolated
heights are established by fitting a surface
through the complete data set.
a) global method
b) local method
c) patchwise method
d) pointwise method
23. The valency of a real stream junction is _____.
a) 2
b) 3
c) 4
d) 8
23. The valency of a real stream junction is _____.
a) 2
b) 3 Valency of a junction network
is number of streams joining at
c) 4 a junction – almost always 3

d) 8

From:
http://www.kumbaya.name/ci2412/M
aterial%20de%20apoyo/Conf%20ArcIn
fo%20en%20SUN/lect-06.html
24. The USGS DEM Standard evaluates the vertical
accuracy of a DEM using the root mean square
error. This value means that approximately
________ of the data falls under the reported
error.
a) 50%
b) 68%
c) 90%
d) 95%
24. The USGS DEM Standard evaluates the vertical
accuracy of a DEM using the root mean square
error. This value means that approximately
________ of the data falls under the reported
error.
a) 50%
b) 68%
c) 90%
d) 95%
25. What is the CE90 (Circular Error 90 percent) for
an orthophoto of 1:12,000 in order to meet map
accuracy standards?
a) 6 meters
b) 10 meters
c) 18 meters
d) 20 meters
NMAS, for maps larger than 1:20,000, no more than 10% of the points
tested shall be in error by more than 1/30” (1/50” for maps ≤ 1:20,000)

25. What is the CE90 (Circular Error 90 percent) for


an orthophoto of 1:12,000 in order to meet map
accuracy standards?
a) 6 meters
b) 10 meters
c) 18 meters
 12,000   1   1' 
   " 
 1   30   12" 
 0.3048 m ft = 10m ( )
d) 20 meters
RECOMMENDED REFERENCES
• Manual of Geographic Information Systems, ASPRS.
• GIS Fundamentals, 6th edition by P. Bolstad
• Manual of Geospatial Science and Technology,2nd edited
by J. Bossler
• Geographical Information Systems, 2nd edition, edited by
P. Longley, M. Goodchild, D. Maguire, and D. Rhind
• An Introduction to Geographical Information System, by I.
Heywood, S. Cornelius, and S. Carver
• Fundamentals of Geographic Information Systems, 4th
edition, by M. DeMers
• Introduction to Geographic Information Systems, 9th
edition, by K. Chang
For Information Contact
Robert Burtch, PS, CP (Ret)

robert.burtch66@gmail.com
rburtch@charter.net

You might also like